NationStates Jolt Archive


Why do people choose not to believe in God?

La Terra di Liberta
05-01-2005, 05:23
It's a question that has puzzled me because when I talk to various people that don't, they seem to have fairly different reasons, so you can't connect the dots. I'm simply curious why because believing or not believing in God is a concious choice we all make unless you're agnostic, so why folks?
Dakini
05-01-2005, 05:36
i'm agnostic.

but when it comes to believing in a god, i really did try. i tried really hard. i just can't bring myself to do it. something doesn't feel right about it and that's all there is to it.

believe me, it would be a lot easier to believe. discussions with my family would be a hell of a lot easier.
Rangerville
05-01-2005, 05:40
Well if people all giving you the same answer helps you connect the dots, it's not going to happen. Like any fundamental question, such as the nature of love, etc. no two people will give you the same answer. I would describe myself as an agnostic, i don't know whether i believe in a higher power or not, though it doesn't mean i don't believe in anything. I just don't need to believe in a God to feel my life is worthwhile, i don't need my morality to be passed on down to me from some entity. I think people should believe in whatever they want though as long as they are not hurting other people. If you find comfort in some higher power, then that's great.
Boonytopia
05-01-2005, 05:40
For me it's more of a case of "why would I believe in god?"

Miracles, an all powerful, all knowing, supernatural creator, eternal paradise/torment. It all seems pretty unrealistic to me.
Mentholyptus
05-01-2005, 05:40
I'm an atheist.

Modern science has given such a successful explanation of just about everything that any necessity for a supreme being is totally gone. Need something to create humans? We've got evolution. Life? Abiogenesis. The Universe? Big Bang and inflationary cosmology, though I'll admit we're still working on that one. Since there's not any kind of logical need for the supernatural (i.e. there's nothing out there that can't be explained in a rational manner-even the stuff science hasn't really gotten laid out yet is expected to become known sooner or later), we don't need any gods. That, plus my own total lack of any experience leading me to believe, is proof enough for me that the Universe gets along just fine without any invisible men in the sky.
MuhOre
05-01-2005, 05:40
People want proof nowadays, simple as that.

But then again we don't have proof on gravity... does that mean gravity doesnt exist? we dont' have proof that something makes up an electron (do we?), yet people believe there to be so.

It all comes down to, what people want to believe for their own moral conciousness. People see their friends getting abortions or being gay w/e, seeing religious people hate them, so... they decide to be as civilized as them, but w/o the G-d.
Corrosive Action
05-01-2005, 05:44
I'm agnostic.

Perhaps your question really is "why don't I believe in your God?".
Anolaconi
05-01-2005, 05:50
I don't believe in any given god as [a] supreme being[s] because there it adds nothing to my life. I don't need religion, and religion doesn't really need me, so why bother? As concepts, though, for personal sprituality, i suppose everone has one god or another.
Jokobee
05-01-2005, 05:52
It's a question that has puzzled me because when I talk to various people that don't, they seem to have fairly different reasons, so you can't connect the dots. I'm simply curious why because believing or not believing in God is a concious choice we all make unless you're agnostic, so why folks?

the question is "why choose to believe in a god?"
Reasonabilityness
05-01-2005, 05:55
Because I don't think he exists.

Simple enough.

I'd very much like there to be an omnibenevolent supreme being. Then I'd be able to pray to him.

Except it seems to me that a God is just wishful thinking.

Seems that way to me, at least. I see no reason to think that a God exists, besides the fact that we want Him to.
Nihilistic Beginners
05-01-2005, 05:56
the question is "why choose to believe in a god?"

Sometimes its not a choice...people have the beilief in god culturally programmed into them and whereas with others its seems that thier brains are hardwired for it.
Roscovia
05-01-2005, 06:06
It all comes down to, what people want to believe for their own moral conciousness. People see their friends getting abortions or being gay w/e, seeing religious people hate them, so... they decide to be as civilized as them, but w/o the G-d.

...Am I the only that didn't get that? Are you saying that being gay is uncivilized?
Cannot think of a name
05-01-2005, 06:07
More or less, it just didn't wash.

There where a few things-the church ridiculing the other beliefs and in the same day telling me about talking bushes on fire and other things that seemed just as ridiculous. It came down to-why is this one more likely than any other? What kind of psychological problem would a being have to have to create a whole world and populate it just so that population could spend all their time telling it how great it is for doing it?

So I became agnostic-no one had any more hold on legitimacy in my view.

Then I started thinking about the tea cup orbiting pluto. Can't prove it's not there, but it just doesn't seem likely. In fact, it seems entirely unlikely. And it seems ridiculous that I would have to accept the possibility of that tea cup no matter how unlikely just because someone became convinced after a night of visions that there was in fact a tea cup orbiting pluto.

So, I still hold the agnostic belief core-we find out what happens to us after we die when we die and if we aren't supposed to be able to 'know god' in the empirical sense then why are there so many people telling us things like he hates Swedes? But-it's all just a tea cup orbiting Pluto. Sure, we could find that there is a tea cup orbiting Pluto, but it's not very likely. I should make my own purpose in life.
Great Agnostica
05-01-2005, 06:09
i'm agnostic.

but when it comes to believing in a god, i really did try. i tried really hard. i just can't bring myself to do it. something doesn't feel right about it and that's all there is to it.

believe me, it would be a lot easier to believe. discussions with my family would be a hell of a lot easier.

That goes for me to. But I am really starting to lean into athieism.
La Terra di Liberta
05-01-2005, 06:10
I'm amazed how it can turn into asking me why I believe in God, which isn't the question. And as far as I know, 2 other religions worship the same God I do and one of the Hindu Gods may be it, so this isn't strictly a christian thing. Maybe I didn't make my first post clear enough, I don't want all the same answers, I'm just looking to see if there are more "common" reasons why people don't.
Roscovia
05-01-2005, 06:12
I'm amazed how it can turn into asking me why I believe in God, which isn't the question. And as far as I know, 2 other religions worship the same God I do and one of the Hindu Gods may be it, so this isn't strictly a christian thing. Maybe I didn't make my first post clear enough, I don't want all the same answers, I'm just looking to see if there are more "common" reasons why people don't.

I think you got a common answer: some people want empirical proof of something before they believe in it, and there is no empirical proof backing up any religion. Ergo, agnosticism and atheism.
Nihilistic Beginners
05-01-2005, 06:12
I'm amazed how it can turn into asking me why I believe in God, which isn't the question. And as far as I know, 2 other religions worship the same God I do and one of the Hindu Gods may be it, so this isn't strictly a christian thing. Maybe I didn't make my first post clear enough, I don't want all the same answers, I'm just looking to see if there are more "common" reasons why people don't.

Because I choose to. Is that clear enough?
Hive Legion
05-01-2005, 06:13
Complex religions were used back in the old days to explain things we didn't understand. We have learned just about everything religion provided a substitute explanation for, and that we have yet to answer is either being answered or will not provide any practical benefit from being answered. Therefore, religion is nothing but fiction mistaken for fact and is no longer neccesary.
Meaning
05-01-2005, 06:14
wat i always wanted to know is who created god nobody ever answered that. thats the main thing but i also remember onces hearing that god did not give angels free will like he gave humans, if this is true that means there is no devil, if there is no devil that means there bad in the world b/c god wishes it. Then i also heard that the devil, death, and hati was the first send to be forever trapped in hell, again if that is true how can there be bad in the world if the bad is trapped? Another thing is Why would a greater being give a crap about us? we're greater then aunts do we care wat they do? and "god" per say is christain/cathlic which saids if u repent and take christ as ur savor then u'll get to heaven, i don't like that b/c it means too think, that a man like stalin or castor can get to heaven if they ask for forgiveness but a man like gohndi can not b/c he does not belive in "god". I would to belive in anything it would be buddism, if ur karma is good and light it shall float up to heaven, if it is dark it will sink and u will go to hell until the bad karma is burned and u are reborn,

the only thing that might make me belive in god is water and ice. well ice is the solid form of water right? will most solid objects tend to sink in there liquid form. ice does not. considering that water is the main source of life and the sea hold a bunch of life then something made ice float in the water that made the living things survivie. or there a reason for this that i do no know of



(NOTE there is another substance that does the same thing its carbon something it is a man made thing so i do not count this)
Pyro Kittens
05-01-2005, 06:20
I can state that god(s) does not exist because there is not scientific evidence for him/her/them and much against what the religons teach. When you stand back and look at it, we created god because that is the kind of creature we are, thus the reason why god (in christanity) is human and male. Please, do not consider this a flame, it is just my oppinion....
Fugee-La
05-01-2005, 06:21
i'm agnostic.

but when it comes to believing in a god, i really did try. i tried really hard. i just can't bring myself to do it. something doesn't feel right about it and that's all there is to it.

believe me, it would be a lot easier to believe. discussions with my family would be a hell of a lot easier.

Same for me...
Gnostikos
05-01-2005, 06:27
I'm amazed how it can turn into asking me why I believe in God, which isn't the question.
It is very hard to answer the reciprocated question, is it not? Your question, though perhaps not intended that way, was quite arrogant. It is asking why anyone believes differently than you do.

And as far as I know, 2 other religions worship the same God I do and one of the Hindu Gods may be it, so this isn't strictly a christian thing.
Hinduism and Christianity are not compatable as far as I know. I do not know a deep amount about either, but saying that one of a pantheon of gods could be the same as your God seems pretty silly to me. Monotheism and polytheism are not the same. And what about things such as heliolatry and selenolatry? And pantheism and panatheism? What about what I am, an agnostic physiolatrist? There are so many different religions, and it really is "just a [Judeo-C]hristian thing." Though I guess Islam and the fringe monotheists could be considered the same; though the non-unitarian Christians believe in the Holy Trinity, which pretty much goes against Judaism and Islam.

Maybe I didn't make my first post clear enough, I don't want all the same answers, I'm just looking to see if there are more "common" reasons why people don't.
Would you like to explain my religious beliefs? As I said earlier, I "practise" agnostic physiolatry. This means that I believe it is not possible to know whether there are any gods or "higher beings". However, I do worpship nature, which is far more beautiful to me than I can imagine anyone seeing an anthropomorphised god as. I found out that my belief and celebration of the cyclical nature is actually similar to druidic beliefs. I know I haven't answered your question, but I just can not find a way to. I would really like to understand how it is possible for you to believe in God. But I know that you'll have just as much trouble explaining your Christianity as I have with my agnostic physiolatry.
R00fletrain
05-01-2005, 06:30
the only thing that might make me belive in god is water and ice. well ice is the solid form of water right? will most solid objects tend to sink in there liquid form. ice does not. considering that water is the main source of life and the sea hold a bunch of life then something made ice float in the water that made the living things survivie. or there a reason for this that i do no know of

(NOTE there is another substance that does the same thing its carbon something it is a man made thing so i do not count this)

that's simply because molecules of h20 spread out when frozen, as a result of hydrogen bonds, i believe..leading to less density and therefore the ability to float..there's a lot of things more complex than that would question my lack of belief in god.
Meaning
05-01-2005, 06:33
that's simply because molecules of h20 spread out when frozen, as a result of hydrogen bonds, i believe..leading to less density and therefore the ability to float...

yes but it's the norm that in a solid the molecules get more compacted?
Glinde Nessroe
05-01-2005, 06:47
I think fear of people that beleive in God (i.e fundamentalist Christians, they really defeat their own purpose), and logical thinkers that look for physical answers that help them.
Dranburg
05-01-2005, 06:51
time to ramble. note, the argument i am asserting will be largely contained and restricted, it is simply to intricate and broad to address with a post.

Why believe in God? I can't articulate the rationality of that to anyone. I can, however, articulate the rationale behind vesting ones beliefs into the metaphysical. This is simply because logic dictates that a dimension of existence (or to remain more neutral; a demension in which forces act) that transcends the physical exists. both by the auspices of ethereal logic and by the laws of fundemental science it is demonstrated that two things of the same definition, cannot produce a thing that does not have properties beyound those of the two objects. this is to say you cannot take two electrons and produce a neutron, or in terms more related to my argument; physical forces cannot create a non-physical (metaphysical) product. this demonstrates that there is a non-physical element present in man; and heres why.

consciousness is inherently metaphysical. science cannot (despite the most ardent efforts on behalf of the most brilliant neuoscientists and psychologists) explain how consciousness is created. This is because the answer is not grounded in the realm of physical science. consciousness - the act of percieving something, the act of FEELING an emotion or sensation, EXPERIENCTING pain, EXPERIENCING physical waves of energy being transposed into the sensation of sound, are inherently NOT PHYSICAL. although in all of these, science can explain how neutransmitters are released when people report pain, sound waves are detected by ones inner ear etc are physical, it cannot explain how these create our EXPERIENCE of them, or in otherwords; consciousness (or atleast a component of conscienscness). this idea of experience or consciousness is not physical, therefore it cannot be produced soley by physical forces. this proves the existance of metaphyics our world. further, it proves humans themselves are in part metaphysical entitites.

In the context of debating the belief in God, this idea does not prove anything, but rather establishes voracity in the belief.
Gnostikos
05-01-2005, 07:01
consciousness is inherently metaphysical. science cannot (despite the most ardent efforts on behalf of the most brilliant neuoscientists and psychologists) explain how consciousness is created. This is because the answer is not grounded in the realm of physical science. consciousness - the act of percieving something, the act of FEELING an emotion or sensation, EXPERIENCTING pain, EXPERIENCING physical waves of energy being transposed into the sensation of sound, are inherently NOT PHYSICAL. although in all of these, science can explain how neutransmitters are released when people report pain, sound waves are detected by ones inner ear etc are physical, it cannot explain how these create our EXPERIENCE of them, or in otherwords; consciousness (or atleast a component of conscienscness).
You see, this is kind of where your argument peters off. Though it really was never that strong. You apparently do not have a firm grasp of neurology, and all the other biological affiliations, such as physiology, the subset of endocrinology, etc., I could go on. There's Richard Dawkins's "hierarchical reductionism" for you. Neurons are actually a pretty conclusive thing for consciousness. Thoughts are created by the electric impulses exchanged between neurons. There are well-known physiological explanations for all emotions. For pain. For vibrations in the air being percieved as sound. None of it is metaphysical now. Granted, we are far from figuring all the ins and outs of any brain, especially the most complex encephalis, the human brain. But we know more than enough to know that we know the jist of what we're going to discover. Metaphysics is outdated in my opinion. Now that our scientists have gone so far in the biochemical fields, physical as well, we no longer need such conceptions as a "soul" or a "spirit".
Dakini
05-01-2005, 07:02
That goes for me to. But I am really starting to lean into athieism.
if i'm leaning towards anything, it's annoyance at both groups.

i once had a christian and an atheist yell at me to pick a side. that might be expected from a christian... but you would think that an atheist would understand a little more that things aren't always as clear cut for everyone...
Gnostikos
05-01-2005, 07:03
yes but it's the norm that in a solid the molecules get more compacted?
The behaviour of hydrogen hydroxide molecules is hardly evidence of God. It is merely the property of that specific compound. Is the fact that any molecules expand or contract proof of God? There are rules to the universe, and that by no means proves God.
Dakini
05-01-2005, 07:04
the only thing that might make me belive in god is water and ice. well ice is the solid form of water right? will most solid objects tend to sink in there liquid form. ice does not. considering that water is the main source of life and the sea hold a bunch of life then something made ice float in the water that made the living things survivie. or there a reason for this that i do no know of
it depends. at different pressures and temperatures ice crystalizes differently.

i fail to see how this is evidence for any kind of god. they have studied how ice forms...
Gnostikos
05-01-2005, 07:05
but you would think that an atheist would understand a little more that things aren't always as clear cut for everyone...
I'm losing faith in atheists. I thought they were better than religionists, but they've been pretty bad sometimes. Stick with agnostics and you'll fare just fine.
Keruvalia
05-01-2005, 07:06
Either way, belief in a god or gods or goddesses or whatever and lack thereof comes down to two words:

Free Will.

People choose. It's why nobody can ever prove definitively that there is or is not a higher power. It's the same reason you can't actually prove to another human being on the planet that you like the taste of chicken.

You either do or you do not.
BLARGistania
05-01-2005, 07:07
People want proof nowadays, simple as that.

But then again we don't have proof on gravity... does that mean gravity doesnt exist? we dont' have proof that something makes up an electron (do we?), yet people believe there to be so.



coughcough. We do. Thats why its not presented as a theory anymore.

We have visable evidence of gravity at work, we have mathematical forumlas to derive it and manipluate it. We can observe its affects on objects. While the term 'gravity' is almost arbitrary, the force it represents is very real and testable.
Sdaeriji
05-01-2005, 07:08
It's a question that has puzzled me because when I talk to various people that don't, they seem to have fairly different reasons, so you can't connect the dots. I'm simply curious why because believing or not believing in God is a concious choice we all make unless you're agnostic, so why folks?

Can't you just happily accept that some people may not believe the same things as you do? Why does anyone have to justify their belief to you?
Commie Catholics
05-01-2005, 07:20
I choose not to believe in God because I have great faith in science. There is a lot science can not currently explain but thats because of the limitations mathematics imposes on it. When (not if, when) we merge General Relativity with Quantum Mechanics we will be able to prove a whole lot more about the universe than we can now.
I don't like it when people base their religion on the fact that water expands when it freezes. Science can explain that easily.
I especially don't like it when certain ignorant people choose to believe in an omnipotent creator with divine influence over all life just because they find the idea of matter suddenly coming into existence a bit farfetched. Anyone who has heard of Heisenberg's Uncertainty Principle knows that this can happen.
Ogiek
05-01-2005, 07:27
It's a question that has puzzled me because when I talk to various people that don't, they seem to have fairly different reasons, so you can't connect the dots. I'm simply curious why because believing or not believing in God is a concious choice we all make unless you're agnostic, so why folks?

I find it interesting that you assume the default would be to believe in god. I think people believe in god, predominantly because they are indoctrinated at an age before they develop rational thought. Yet, in spite of that indoctrination people with no religion, atheists, agnostics, freethinkers, humanists, secularists, etc. make up the third largest group in the world, only behind Christians and Muslims.

I wonder what would happen if we were one day visited by aliens and they had absolutely no idea what we were referring to when we talked of god and religion?
Freemanistan
05-01-2005, 07:29
OK, big problems with the Dranburg's post...first, WTF is "ethereal logic"? And, as for "logic dictating," in spite of the best efforts of wise men for thousands years looking at the sky, man was unable to understand that the earth was spherical until we sailed around it. Logic doesn't dictate anything other than "we don't yet understand." Just because we don't yet fully know the mechanism for consciousness (we do know quite a bit already) doesn't mean it isn't physical. Further, the fact that adjusting chemicals in one's body can dramatically effect one's mood or sense of reality suggests that conciousness is indeed a state arising from the physical (electro-chemical) properties of the brain.

I don't believe in god for several reasons. First, because the moral implications of god's existence are appaling on so many levels, that to believe in any god would thrust me into a depression so deep I might be driven to suicide. No, it is far better to attribute the events in the universe to the divergent desires of individuals and the random chance associated with weather, disease, geologic events and so forth. Also, to believe in god is disempowering, since it removes from you the responsibility and fulfillment that comes from creating meaning in your own life. Further, knowing that life is finite imparts a value to every day. If you really believe that you will live forever in heaven, and that heaven is better than here, why bother? Why not die a martyr, or smoke your guts out, as long as you go to confession or follow the commandments, you're good. Really, if you could never run out of money, would yo give a shit how you spent it?

Lastly, there is simply no way to KNOW whether god exists or not, but I do know that he/she/it has never spoken to me or made a godly presence known to me, or performed a miracle on my behalf, or answered a prayer, or any number of other things that might encourage me to believe that a diety exist who gives two shits what I do on a daily basis. I do find nature beautiful and awe inspiring, and it fills me with reverence for whatever brought it about, you can call it creation if you like, but it is beyond my understanding as of now. That doesn't prove it wasn't a natural phenomenon, or a random accident, nor does it prove it to be an act of providence. What it comes down to is, I don't know. But I do know that organized religion is directly responsible for the deaths of millions, and so much mental anguish and false hope, it is a crime. So too is state worship.

So I will choose to worship nothing, and to live for myself, cherishing every day until the very last one. If there is a god, he is ultimately responsible for my being, and he can't hold it against his own creation that I failed to grasp his true nature with the faculties I was given by him to examine the evidence before me in the world he built. If he does, he is a cruel and unjust god, unworthy of my devotion anyway.
Eridanus
05-01-2005, 07:31
Well, unless you're a convert, or you jsut find some reason to believe in god, you grow up believing in him, and the thought of not jsut never really occurs to you. Not believing in god is the same, jsut reversed.
Boonytopia
05-01-2005, 07:32
I wonder what would happen if we were one day visited by aliens and they had absolutely no idea what we were referring to when we talked of god and religion?[/QUOTE]

Very interesting thought. I was brought up in a totally non-religious household, never went to church as a child, etc. I find it odd that so many people have such a fervent belief in nothing (as I percieve it). To me, a lack of god/religion is the default.
Commie Catholics
05-01-2005, 07:48
I myself am a proud atheist. Although i can see why people choose to believe God. I am a String theorist. Those that know a bit about string theory would probably agree that it is one of the most wonderful ideas ever though of. They would also know that it has one fatal flaw. It can't be disproved by any physical observation. I can see where christians are coming from but string theory has a lot more scientific merit than God does.
UpwardThrust
05-01-2005, 07:52
i'm agnostic.

but when it comes to believing in a god, i really did try. i tried really hard. i just can't bring myself to do it. something doesn't feel right about it and that's all there is to it.

believe me, it would be a lot easier to believe. discussions with my family would be a hell of a lot easier.
Same here

got to love sitting in a corner being quiet while your family goes off on something or other.

And you got to love when you talk to christian friends that bring up a standard arguement... and you bring up something as simple as levidicus ... they look at you like you are a heritic (thats when you can tell they never read the whole bible only the parts that support their arguements)

It gets awkward being agnostic but knowing more then my christian friends (I have to cite verse and chappter or they wont believe it exists)

All in all would be easier to be in the croud if I could
Narazene
05-01-2005, 08:41
I'm an athiest.

I stopped believing in a God because of all the crap I've seen in my life. I found it very difficult to believe there was some sort of higher power who could stop all this horrible stuff going on, but didn't. I struggled with this for a long time before I came to the conclusion that, in my life at least, there was no God or Satan, nor was there a Heavan or a Hell. There was no reason for it either.
Soviet Narco State
05-01-2005, 08:57
church is boring.
Branin
05-01-2005, 09:05
Why do people choose not to believe in God?

Because they either don't want to, or feel they need evidence and there isn't enough. That is what I have seen.

Beliving and proud of it. (Yah, Yah, shut up, I know I'm supposed to be a liberal)
Kulkungrad
05-01-2005, 09:20
Futurama put it best:
You know you've done things right, when nobody is certain you've done anything at all.

I see the miracles in my life as well as all the tragedies. 9/11 was a horrible evil that occured to innocents, followed shortly by immense outpouring of love, emotions, and sympathy. The tsunami killed over a hundred thousand people (might be another hundred thousand after the disease epidemic is over), and it too is being followed by an almost worldwide outpouring of love and support (Charity contributions by Americans for the tsunami victims are close to the massive American Government contribution).

My life isn't perfect (heck it's barely good) despite my faith and I can understand those who stop believing because of their personal experiences (as sad as I believe it is). I don't count on God for anything major. I don't pray for wealth or fame in life because those only last on this world. When you're dead it's all gone. I look forward to my death. I'm not going to kill myself or put myself in a situation of extreme danger, but I am content to live my life with all it's ups and downs until the very end, whether it be in a hospital a hundred years from now, or the day after tomorrow in my car as I drive to work.

In my personal experience, I find that most people who don't believe in a God believe that God must do everything for us and choose not to believe because they never had "a reason to" as if God was supposed to swoop down and give them a wad of money or suddenly come to their arguing parents and plant love in their hearts so they stop their divorce.

Alot of people (even one post in this thread) believe that people have to be indoctrinated to have faith in a religion. Yet how do they explain those who actually GAIN a faith in a religion? Those who never believed in anything, whether agnostic or aethiests, suddenly encounter some form of miracle or realization that there might be a higher power?

Edit: I see I began preaching below. Ignore if you want.

Life is God's greatest gift because we actually must challenge ourselves and learn. Would heaven really be heaven if we all started there? Would it really be so great to be in heaven if we just existed there, everything we ever wanted being catered to just by thinking it? Sounds good here, doesn't it? I view life as a trial and a journey. You are born and your existence alone is a miracle. That from two halves of a single set of chromosomes, cells replicate rapidly and create your brain which can be molded and be so different from other people. You live your life taking in all sorts of information and experiences and if you possess a true faith at the end of the journey, you're rewarded with an oasis at the end of the long road.
Greedy Pig
05-01-2005, 09:35
church is boring.

Then you haven't been to my church :p
Greedy Pig
05-01-2005, 09:41
I think people choose not to believe in God (Agnostics, beleive in God, but not a per se religion), because there's so much religion out there, and each believe in different things (more or less). Probably because there's so many bad hats in every religion, that makes it hard to commit.

Those choose not to believe there is a God (Atheist), I guess they believe that he doesn't exist.

To me, I'm A Christian. If not I'll be a agnostic (too much weird shit happend in my life to believe there isn't a God), but I'm a Christian because it's doctrines and message of redemption really speaks to me more than other religions.
Illich Jackal
05-01-2005, 10:14
second generation atheist. I just never got indoctrinated by religions.
Subjective Pragmatism
05-01-2005, 10:41
Well, I was brought up in a very religious family, but as I got older(12), I saw a lot of flaws (in my opinion) with Christianity...and religion in general.

I also was able to rationalize why religions were created...and none of those reasons constitutes its existence.
The Alma Mater
05-01-2005, 10:53
I believe in the possibility of a god, multiple gods or beings that are sufficiently godlike to claim such a title.

I see however no reason to worship them. Respect him/them since he/they can do things I cannot: yes. But I don't worship plumbers either.
Erehwon Forest
05-01-2005, 10:56
Although my parents are both (sort of) christians, I wasn't exactly indoctrinated as a child. In my childhood, god(s) was(/were) just one possible theory, my parents allowed me to make my own choices about the matter. By the time I developed the ability to really think for myself, religion just seemed like a really silly story people get so horribly worked up over.

At 14 I took a "course" that all Finns have to take to be members of the evangelical-lutheran church, where we just went through some basics from the bible and learned some prayersm etc. This very strongly enforced the "silly story" line of thinking. I realized I quite simply have zero faith in any god whatsoever.

Until I was maybe 18 I figured a complete lack of belief in anything "superhuman" made me an atheist. Then agnosticism was discussed in, I'm not sure, maybe some high school philosophy group? That just fitted me perfectly.
Snorklenork
05-01-2005, 11:48
As far as I can tell, people believe in God (or other type) for more or less two reasons. One is the explain the unexplainable. The biggest example of that is to explain the existence of the universe (i.e. God created it). To me this is simply overcomplicating things. They can happily accept God existing without being created, but not the universe. I just feel that one's as good as the other, except one doesn't add a superfluous, untestable layer on my beliefs.

The second reason often is for some sort of moral support. That is, they want to feel that their moral code is based in something great and absolute. That is, they can be right and everyone else who says otherwise is wrong. Well, that may lend comfort to one's life, and give someone some sort of moral righteousness, but that doesn't make them right. It's not good enough to believe in something because of personal convenience.

That's my rationalization of why I don't believe in god: it doesn't fill any holes for me. Of course, it may be that there are other reasons that I don't believe in a god, but in that case you'd be better off asking a psychologist (maybe).
Nihilistic Beginners
05-01-2005, 11:53
I actually don't think there is much choice involved, there is alot of evidence that people are genetically pre-disposed to believing in the supernatural and some are not, but the vast majority of people are hardwired to believe in the supernatural, and those who are not are in the minority....like some kind of anomaly.
Mekonia
05-01-2005, 11:55
Because if they do/did believe in God, they feel
1)their chruch/religion is a pile of shit, that is just complete propaganda.
2) Their Mammies don't force them to go to Chruch anymore, all the years of get out of that bed and get your ass in to mass have gone against religion.
3) In the 21st century there are more plausible scientific explanations to who created the world then the 7day variety you can find in any good religious publications house!
4) They believe God has forsaken them. What kind of God would allow world war, terrorism and tsunamis?
5)Because there is no God it was all pretend made up by some wanna be King who wanted to be remembered for eternity cos he did nothing with his time on the throne!

Choice is yours!
Bottle
05-01-2005, 17:08
People want proof nowadays, simple as that.

But then again we don't have proof on gravity
yes, we do.

we dont' have proof that something makes up an electron (do we?)
yes, we do.


It all comes down to, what people want to believe for their own moral conciousness. People see their friends getting abortions or being gay w/e, seeing religious people hate them, so... they decide to be as civilized as them, but w/o the G-d.
so you are saying people choose to believe in whichever God (or lack of God) fits with the morality they already have chosen?
Drunk commies
05-01-2005, 17:10
People want proof nowadays, simple as that.

But then again we don't have proof on gravity... does that mean gravity doesnt exist? we dont' have proof that something makes up an electron (do we?), yet people believe there to be so.

It all comes down to, what people want to believe for their own moral conciousness. People see their friends getting abortions or being gay w/e, seeing religious people hate them, so... they decide to be as civilized as them, but w/o the G-d.
We can observe gravity's effects on the real world. The same can't be said for god.
Drunk commies
05-01-2005, 17:12
wat i always wanted to know is who created god nobody ever answered that. thats the main thing but i also remember onces hearing that god did not give angels free will like he gave humans, if this is true that means there is no devil, if there is no devil that means there bad in the world b/c god wishes it. Then i also heard that the devil, death, and hati was the first send to be forever trapped in hell, again if that is true how can there be bad in the world if the bad is trapped? Another thing is Why would a greater being give a crap about us? we're greater then aunts do we care wat they do? and "god" per say is christain/cathlic which saids if u repent and take christ as ur savor then u'll get to heaven, i don't like that b/c it means too think, that a man like stalin or castor can get to heaven if they ask for forgiveness but a man like gohndi can not b/c he does not belive in "god". I would to belive in anything it would be buddism, if ur karma is good and light it shall float up to heaven, if it is dark it will sink and u will go to hell until the bad karma is burned and u are reborn,

the only thing that might make me belive in god is water and ice. well ice is the solid form of water right? will most solid objects tend to sink in there liquid form. ice does not. considering that water is the main source of life and the sea hold a bunch of life then something made ice float in the water that made the living things survivie. or there a reason for this that i do no know of



(NOTE there is another substance that does the same thing its carbon something it is a man made thing so i do not count this)
Ice is no great mystery. It's molecular structure causes it to arrange itself into crystals of a certain shape when it solidifies. that shape takes up more space than the same mass of liquid water.
Drunk commies
05-01-2005, 17:14
yes but it's the norm that in a solid the molecules get more compacted?
Usually yes. It depends on the shape of the molecule and whether it's polar or not (has a positively charged spot and a negatively charged spot)
Pythagosaurus
05-01-2005, 17:26
It's a question that has puzzled me because when I talk to various people that don't, they seem to have fairly different reasons, so you can't connect the dots. I'm simply curious why because believing or not believing in God is a concious choice we all make unless you're agnostic, so why folks?
Believing is not a choice. Saying that you believe is a choice.

For example, I make the following proposition. A million people tell you that if you believe that there are naturally occurring pink elephants, then you will win the lottery. If you don't, your house will burn down. There's no particular disadvantage to believing in pink elephants. Thus, a simple cost-benefit analysis indicates that everybody should believe in pink elephants. Now, do you believe in pink elephants? I didn't think so. Why would you make such a choice? Do you want your house to burn down?

I have made a comparison between God and pink elephants. You might object that the concept of pink elephants is ludicrous. On what grounds would you base that? There are black swans in Australia. Lots of new animals were discovered on Galapagos. There could easily be pink elephants somewhere. You still don't believe in them, do you? After a million people have told you that you'd win the lottery if you just believed, you would still rather have your house burn down. Well, that's quite irrational.

Why would you make such a choice?
Grave_n_idle
05-01-2005, 17:33
It's a question that has puzzled me because when I talk to various people that don't, they seem to have fairly different reasons, so you can't connect the dots. I'm simply curious why because believing or not believing in God is a concious choice we all make unless you're agnostic, so why folks?

I think you may be labouring under a misapprehension... if you seriously believe that 'not believing in god' is a choice.

As an atheist, I realised I did not believe in god. Prior to that point I had been (nominally) Anglican... but the whole thing was getting fuzzier the more I studied it, and it didn't match with observable reality.

Eventually, I found that I no longer believed, and that is what 'makes' em an Atheist.

I didn't say, "I've had enough of this, bye bye god, I'm not believing in you anymore"... there was no 'choice' - it just doesn't make any sense to me.

Let me reverse it - you claim that "believing or not believing in God is a concious choice we all make"... does that mean that, at some point, you DECIDED to stop being a godless heathen, and CHOSE god? (Which would make him something of a consolation prize, surely?).

We are born believing in no gods. We are 'educated' with religion, and taught (while still young, and therefore trusting) that 'god' (ir gods) is 'true'.
UpwardThrust
05-01-2005, 17:40
I think you may be labouring under a misapprehension... if you seriously believe that 'not believing in god' is a choice.

As an atheist, I realised I did not believe in god. Prior to that point I had been (nominally) Anglican... but the whole thing was getting fuzzier the more I studied it, and it didn't match with observable reality.

Eventually, I found that I no longer believed, and that is what 'makes' em an Atheist.

I didn't say, "I've had enough of this, bye bye god, I'm not believing in you anymore"... there was no 'choice' - it just doesn't make any sense to me.

Let me reverse it - you claim that "believing or not believing in God is a concious choice we all make"... does that mean that, at some point, you DECIDED to stop being a godless heathen, and CHOSE god? (Which would make him something of a consolation prize, surely?).

We are born believing in no gods. We are 'educated' with religion, and taught (while still young, and therefore trusting) that 'god' (ir gods) is 'true'.


Agreed … I read and studied (grew up roman catholic) and my opinions changed … mostly a lack of belief in the ability of another human to interpret correctly the will of god … then a realization that the bible itself was just a document (highly flawed) and was an attempt at putting to words the beliefs of god (or humans think is the will of god)

Add to that translation and contextual errors …

I just realized that was all too flaky for me to but a belief in… at that I would have stopped at something like deism. But as I studied things more things just did not mesh.


So here I am … about 80 percent agnostic 20 soft atheism…. So I am fairly sure we don’t know right now if he exists or if we can tell right now … with a slight leaning towards just not believing in him
(dont know if this came out right ... but yeah hard to put a COMPLETE philosophy into words)
Sirius Zero
05-01-2005, 17:53
It's a question that has puzzled me because when I talk to various people that don't, they seem to have fairly different reasons, so you can't connect the dots. I'm simply curious why because believing or not believing in God is a concious choice we all make unless you're agnostic, so why folks?

Why should I? I've never seen a god. Have you? When a god comes down and says, "Hey, you over there with the long hair and the motorcycle boots! Kneel and worship me!" I'll accept that gods exist. And then I'll look for a weapon that can kill them.
Grave_n_idle
05-01-2005, 17:54
Agreed … I read and studied (grew up roman catholic) and my opinions changed … mostly a lack of belief in the ability of another human to interpret correctly the will of god … then a realization that the bible itself was just a document (highly flawed) and was an attempt at putting to words the beliefs of god (or humans think is the will of god)

Add to that translation and contextual errors …

I just realized that was all too flaky for me to but a belief in… at that I would have stopped at something like deism. But as I studied things more things just did not mesh.


So here I am … about 80 percent agnostic 20 soft atheism…. So I am fairly sure we don’t know right now if he exists or if we can tell right now … with a slight leaning towards just not believing in him
(dont know if this came out right ... but yeah hard to put a COMPLETE philosophy into words)

I think you managed to sum it up pretty well.

You aren't SURE if there is a god, or not - but you are INCLINED (slightly) towards the 'not' answer.

:)
UpwardThrust
05-01-2005, 17:57
I think you managed to sum it up pretty well.

You aren't SURE if there is a god, or not - but you are INCLINED (slightly) towards the 'not' answer.

:)
Exactly … I know there is almost no proof … but I tend to thing scientifically so generally I go with the theory of proving something true rather then false so that leads me to lean towards “not” (would be all the way there if I had some faith in being able to prove anything at all lol)
GoatZone
05-01-2005, 17:58
Maybe this Stephen Roberts quote will help you understand why some people do not believe in God -

"I contend that we are both atheists. I just believe in one fewer god than you do. When you understand why you dismiss all the other possible gods, you will understand why I dismiss yours."
Grave_n_idle
05-01-2005, 18:01
Exactly … I know there is almost no proof … but I tend to thing scientifically so generally I go with the theory of proving something true rather then false so that leads me to lean towards “not” (would be all the way there if I had some faith in being able to prove anything at all lol)

I don't 'believe' anything... to be honest.

I accept certain patterns emerge, and that, once a pattern emerges, it seems to continue....

Thus, I suspect, from observation, that the sun will very likely rise tomorrow.

But do I 'believe' it? Or do I just have a good 'idea' that it is likely?
UpwardThrust
05-01-2005, 18:07
I don't 'believe' anything... to be honest.

I accept certain patterns emerge, and that, once a pattern emerges, it seems to continue....

Thus, I suspect, from observation, that the sun will very likely rise tomorrow.

But do I 'believe' it? Or do I just have a good 'idea' that it is likely?
Yeah I have seen your argument before that’s why I think I could be soft atheist more then agnostic (unsure) but I know I couldn’t be hard atheist
Grave_n_idle
05-01-2005, 18:20
Yeah I have seen your argument before that’s why I think I could be soft atheist more then agnostic (unsure) but I know I couldn’t be hard atheist

That's because 'Hard Atheism' is just as hard a vantage to justify as ANY of the religious denominations...

You theoretically have to have evidence that there is NO god, which is as hard (or easy) to accomplish as evidence that there IS a god.

My observations lead me to the conclusion that there MOST LIKELY is no god (or nothing we CONCEIVE as a 'god') - but I'm not going to bet my next breath on it being a VERIFIABLE fact.
UpwardThrust
05-01-2005, 18:23
That's because 'Hard Atheism' is just as hard a vantage to justify as ANY of the religious denominations...

You theoretically have to have evidence that there is NO god, which is as hard (or easy) to accomplish as evidence that there IS a god.

My observations lead me to the conclusion that there MOST LIKELY is no god (or nothing we CONCEIVE as a 'god') - but I'm not going to bet my next breath on it being a VERIFIABLE fact.
Yup ... same ... on that grey agnostic/soft atheist line
La Terra di Liberta
05-01-2005, 18:25
I'm amazed how much shit I've gotten for asking this question and how I sound arrogant. Let me explain: people seem to constantly question why people believe in God, they say it's ridiculous, absured, etc and when people try and defend their faith, sometimes they are minimized for it. I'm not say Christians don't do this with athiest but my question was simply a bit of a counter to the question "Why do you choose to believe in God?". So next time someone asks that, are they being arrogant and selfish? And if you don't believe in him, for whatever reason, it is a choice. You may look at whats going on in the world and say "With all the death and disease in the world, there cannot be a God because if he loves us so much, why would he allow this to happen?" Someone else may look at it though that things happen in nature and decisions are made by man that result in this and God has given us free will so he won't interfere in these happenings. Is that a choice and if not, what is it?
La Terra di Liberta
05-01-2005, 18:26
Why should I? I've never seen a god. Have you? When a god comes down and says, "Hey, you over there with the long hair and the motorcycle boots! Kneel and worship me!" I'll accept that gods exist. And then I'll look for a weapon that can kill them.



Just because you haven't seen something/one doesn't mean always it/they don't exist.
Pythagosaurus
05-01-2005, 18:29
I'm amazed how much shit I've gotten for asking this question and how I sound arrogant. Let me explain: people seem to constantly question why people believe in God, they say it's ridiculous, absured, etc and when people try and defend their faith, sometimes they are minimized for it. I'm not say Christians don't do this with athiest but my question was simply a bit of a counter to the question "Why do you choose to believe in God?". So next time someone asks that, are they being arrogant and selfish? And if you don't believe in him, for whatever reason, it is a choice. You may look at whats going on in the world and say "With all the death and disease in the world, there cannot be a God because if he loves us so much, why would he allow this to happen?" Someone else may look at it though that things happen in nature and decisions are made by man that result in this and God has given us free will so he won't interfere in these happenings. Is that a choice and if not, what is it?
It's not the question you asked. It's the way you asked it.

Do you believe in pink elephants yet?
Drunk commies
05-01-2005, 18:31
Just because you haven't seen something/one doesn't mean always it/they don't exist.
Doesn't mean they do either. And when the thing that's supposed to exist has such extraordinary properties (no end and no beginning, can create universes at will, creates all forms of life, etc.) you'd be justified in asking for some evidence.
La Terra di Liberta
05-01-2005, 18:34
It's not the question you asked. It's the way you asked it.

Do you believe in pink elephants yet?




What is the "way" I asked it? People are constantly asked why they believe in God, so why is it so wrong to ask why people don't? The super natural and the physical world are two different things. Pink elephants are proven not to exist, it hasn't been proven God doesn't exist.
PIcaRDMPCia
05-01-2005, 18:36
It's a question that has puzzled me because when I talk to various people that don't, they seem to have fairly different reasons, so you can't connect the dots. I'm simply curious why because believing or not believing in God is a concious choice we all make unless you're agnostic, so why folks?
Lack of proof. I need proof before I believe in anything.
La Terra di Liberta
05-01-2005, 18:37
Lack of proof. I need proof before I believe in anything.



Do you believe aliens/ufos exist?
LindsayGilroy
05-01-2005, 18:40
I was raised to believe that there is no god. I have attended cofe and catholic church and read around Islam and have maintained my belief that there isnt a god. For me I've always believed in science.
UpwardThrust
05-01-2005, 18:41
Do you believe aliens/ufos exist?
No but there is a fairly decent statistical probability that life in SOME form exists elsewhere
Pythagosaurus
05-01-2005, 18:45
What is the "way" I asked it? People are constantly asked why they believe in God, so why is it so wrong to ask why people don't? The super natural and the physical world are two different things. Pink elephants are proven not to exist, it hasn't been proven God doesn't exist.
You asked it in an obviously biased manner, and that makes people defensive. People who ask you why you believe in God probably are being just as arrogant. That doesn't make it right. That doesn't mean that you should respond in kind.

There is no proof that pink elephants don't exist. Somebody who has never heard of Australia would think that swans can only be white. Before Galapagos was found, there were plenty of things that "didn't exist" that now do.

So, why don't you believe in pink elephants?
Drunk commies
05-01-2005, 18:45
Do you believe aliens/ufos exist?
I know this wasn't directed to me, but I'm bored and I'm going to put my two cents in.

Aliens, yeah, it's possible, but I don't have enough evidence to say one way or another. Still, there's life on this planet, so I find life on another planet not to be a very extraordinary claim. On aliens I'll say it's likely but I don't know.

UFOs? Well in the strictest definition of UFO, yes. People see flying objects that they can't identify. That's an unidentified flying object. If you mean spacecraft piloted by an alien race, No, I don't beleive. Simply because faster than light travel is impossible. That makes the claim that aliens have visited earth an extraordinary claim. I need some hard evidence before I'll beleive in that.
La Terra di Liberta
05-01-2005, 18:54
You asked it in an obviously biased manner, and that makes people defensive. People who ask you why you believe in God probably are being just as arrogant. That doesn't make it right. That doesn't mean that you should respond in kind.

There is no proof that pink elephants don't exist. Somebody who has never heard of Australia would think that swans can only be white. Before Galapagos was found, there were plenty of things that "didn't exist" that now do.

So, why don't you believe in pink elephants?




I didn't know asking someone why they believe something was arrogant. Anyway, I was going to us the example of countries but then I thought people might say the scientific proof is there, so I swayed to something a little less clear. I don't believe in pink elephants because I never heard of anyone whos found one or anyone who believes they exist. If I went to my city zoo and one of the biologists or whatever they are called there if they do, I doubt I'd get a yes. I can see you saying the same thing about God but I'll wait just to make sure before I start yo defend it.
La Terra di Liberta
05-01-2005, 18:54
I know this wasn't directed to me, but I'm bored and I'm going to put my two cents in.

Aliens, yeah, it's possible, but I don't have enough evidence to say one way or another. Still, there's life on this planet, so I find life on another planet not to be a very extraordinary claim. On aliens I'll say it's likely but I don't know.

UFOs? Well in the strictest definition of UFO, yes. People see flying objects that they can't identify. That's an unidentified flying object. If you mean spacecraft piloted by an alien race, No, I don't beleive. Simply because faster than light travel is impossible. That makes the claim that aliens have visited earth an extraordinary claim. I need some hard evidence before I'll beleive in that.



I believe they exist, although for different reasons.
PIcaRDMPCia
05-01-2005, 18:59
Do you believe aliens/ufos exist?
I believe that, with the stastics at hand and with what we know of the universe, the odds of life existing on other planets besides Earth are very, very high.
GoatZone
05-01-2005, 19:04
... it hasn't been proven God doesn't exist.
But how can you prove a negative?

If I tell you there are invisible pink unicorns at the bottom of my garden can you prove they do not exist?

Evidence is key. While I could counter anything you might say against the existence of IPUs, I could not supply any hard evidence that they do exist.

It is not the atheist who needs to provide evidence of the non-existence of God, it is the believer who needs to show evidence that God exists.
Grave_n_idle
05-01-2005, 19:05
What is the "way" I asked it? People are constantly asked why they believe in God, so why is it so wrong to ask why people don't? The super natural and the physical world are two different things. Pink elephants are proven not to exist, it hasn't been proven God doesn't exist.

I think half the problem was, that you imply that it is a 'choice'.... thereby, also implying that people decide to 'turn their backs' on god.

As I stated... it didn't make sense to me, I found out I didn't believe it anymore... I didn't 'chose' to do that.

Did you 'chose' to be christian? Or did you find yourself convinced by the evidence you saw?

By asserting christianity as 'right', you denigrate all non-christians, but especially the atheist. Effectively, you imply that they have 'chosen' to be, somehow, 'deficient'.

By the way - Pink Elephants have NEVER been conclusively proved not to exist... which means there is the SAME amount of verifiable evidence for pink elephants, as there is for 'god'.
Drunk commies
05-01-2005, 19:06
But how can you prove a negative?

If I tell you there are invisible pink unicorns at the bottom of my garden can you prove they do not exist?

Evidence is key. While I could counter anything you might say against the existence of IPUs, I could not supply any hard evidence that they do exist.

It is not the atheist who needs to provide evidence of the non-existence of God, it is the believer who needs to show evidence that God exists.
I can prove that there are no invisible pink unicorns. Pink means that they reflect that wavelength of light. Invisible means light goes right through them. Since no object could have both those properties, your invisible pink unicorns cannot exist.
Pythagosaurus
05-01-2005, 19:06
I didn't know asking someone why they believe something was arrogant. Anyway, I was going to us the example of countries but then I thought people might say the scientific proof is there, so I swayed to something a little less clear. I don't believe in pink elephants because I never heard of anyone whos found one or anyone who believes they exist. If I went to my city zoo and one of the biologists or whatever they are called there if they do, I doubt I'd get a yes. I can see you saying the same thing about God but I'll wait just to make sure before I start yo defend it.
Consider some of the people who appear arrogant when they ask you why you believe in God. Are there any? If not, then you're very lucky. Do you think that those people could have changed their wordings so as not to appear arrogant? Now, turn the coin around. When people backlash, it isn't because of the question. It's because of the phrasing.

That's sort of the point. However, a zoologist or biologist will also admit that he doesn't know a thing about geography. He will admit that he has no idea whether or not the entire world has been explored. He will admit that he doesn't know what animals exist on other planets. He will not say that it is impossible for pink elephants to exist.

Now, consider the millions of people who believe in God. What do they know about God? They've studied books that are supposedly the word of God. They listened to other people who supposedly know the word of God.

Why do you believe them? What makes a million people more believable than one? How about if two people told you my story about pink elephants? What if a hundred people told you my story about pink elephants? How many people do you have to meet in order for you to believe something? And how many of them have to have evidence?

What makes God more believable, as a concept, than pink elephants?
La Terra di Liberta
05-01-2005, 19:08
I think half the problem was, that you imply that it is a 'choice'.... thereby, also implying that people decide to 'turn their backs' on god.

As I stated... it didn't make sense to me, I found out I didn't believe it anymore... I didn't 'chose' to do that.

Did you 'chose' to be christian? Or did you find yourself convinced by the evidence you saw?

By asserting christianity as 'right', you denigrate all non-christians, but especially the atheist. Effectively, you imply that they have 'chosen' to be, somehow, 'deficient'.

By the way - Pink Elephants have NEVER been conclusively proved not to exist... which means there is the SAME amount of verifiable evidence for pink elephants, as there is for 'god'.



You bring up a good point but heres a thought: two people can look at a painting and see two completely different things. Is either wrong? Similarly with God, I see things in the world that convince me that he exists but someone else may see the same thing and say that doesn't prove he does. Just a thought.
Grave_n_idle
05-01-2005, 19:11
You bring up a good point but heres a thought: two people can look at a painting and see two completely different things. Is either wrong? Similarly with God, I see things in the world that convince me that he exists but someone else may see the same thing and say that doesn't prove he does. Just a thought.

Okay - we are standing, you and I, looking at some kind of Rorshache blot of modern art.

It's very pretty, we both agree.

You see the face of an angel, in the blot.

I see the face of a long-lost-friend.

Maybe we are both right, maybe neither of us sees what the artist intended.

Maybe your 'angel' is the same thing I see when I see my 'long-lost-friend'.

The point is - looking at that same blot... which of us CHOSES to see a different image to the other?
La Terra di Liberta
05-01-2005, 19:13
Consider some of the people who appear arrogant when they ask you why you believe in God. Are there any? If not, then you're very lucky. Do you think that those people could have changed their wordings so as not to appear arrogant? Now, turn the coin around. When people backlash, it isn't because of the question. It's because of the phrasing.

That's sort of the point. However, a zoologist or biologist will also admit that he doesn't know a thing about geography. He will admit that he has no idea whether or not the entire world has been explored. He will admit that he doesn't know what animals exist on other planets. He will not say that it is impossible for pink elephants to exist.

Now, consider the millions of people who believe in God. What do they know about God? They've studied books that are supposedly the word of God. They listened to other people who supposedly know the word of God.

Why do you believe them? What makes a million people more believable than one? How about if two people told you my story about pink elephants? What if a hundred people told you my story about pink elephants? How many people do you have to meet in order for you to believe something? And how many of them have to have evidence?

What makes God more believable, as a concept, than pink elephants?



You mention how a question is worded to sound arrogant but anyway, the belief in God is more believable because it has exist since very early in the history of man and has continued down throught the generations. If it were such a weak theory, it wouldn't have last very long but it has. Maybe pink elephants live at the bottom of the sea, although give the current elephants live on land, thats highly unlikely or maybe an small island in the South Pacific , Asia or Africa. that has not yet been visited. You can give a bit on either one.
GoatZone
05-01-2005, 19:14
Invisible means light goes right through them.
Nope, invisible means impossible to see. You're thinking "transparent".


(edit - put quotes around "transparent")
La Terra di Liberta
05-01-2005, 19:14
Okay - we are standing, you and I, looking at some kind of Rorshache blot of modern art.

It's very pretty, we both agree.

You see the face of an angel, in the blot.

I see the face of a long-lost-friend.

Maybe we are both right, maybe neither of us sees what the artist intended.

Maybe your 'angel' is the same thing I see when I see my 'long-lost-friend'.

The point is - looking at that same blot... which of us CHOSES to see a different image to the other?



So you admit its a choice then? Or is it based on experiences in our lives that shape how we see it?
LindsayGilroy
05-01-2005, 19:15
You mention how a question is worded to sound arrogant but anyway, the belief in God is more believable because it has exist since very early in the history of man and has continued down throught the generations. If it were such a weak theory, it wouldn't have last very long but it has. Maybe pink elephants live at the bottom of the sea, although give the current elephants live on land, thats highly unlikely or maybe an small island in the South Pacific , Asia or Africa. that has not yet been visited. You can give a bit on either one.
Actually the belief in one particular religion has been since the dawn of man. Since the dawn of man various gods have been idolised and there are so many religions to choose from.
Drunk commies
05-01-2005, 19:15
Nope, invisible means impossible to see. You're thinking transparent.
Invisibility is identical to perfect transparency, no?
Frangland
05-01-2005, 19:18
This is an obvious answer, but many people simply choose to believe only what they can see.
GoatZone
05-01-2005, 19:20
Not in the case of IPUs.

To the non-believer they are invisible. If you believe in IPUs then you can see them - and they appear to be pink. They have amazing powers when it comes to the manipulation of visible light.
Drunk commies
05-01-2005, 19:21
Not in the case of IPUs.

To the non-believer they are invisible. If you believe in IPUs then you can see them - and they appear to be pink. They have amazing powers when it comes to the manipulation of visible light.
Ok
Pythagosaurus
05-01-2005, 19:21
You mention how a question is worded to sound arrogant but anyway, the belief in God is more believable because it has exist since very early in the history of man and has continued down throught the generations. If it were such a weak theory, it wouldn't have last very long but it has. Maybe pink elephants live at the bottom of the sea, although give the current elephants live on land, thats highly unlikely or maybe an small island in the South Pacific , Asia or Africa. that has not yet been visited. You can give a bit on either one.
So you do believe in pink elephants?

O.K. Billions of people. We've been through this. How many does it take?

The belief that the Earth was flat lasted for generations upon generations. For some reason it had to be proven otherwise. Why is that a better default?
La Terra di Liberta
05-01-2005, 19:22
Stay on topic people, unless invisible or transparent relates to God.
Slinao
05-01-2005, 19:25
I belive in G.O.D. and N.O.A.H.


I think that we all come from an alien project that was titled, Nebulons of Anamated Herpes aka N.O.A.H.

Its found right there in the bible. It talks about a great flood and how the earth was destroyed and that he had to take 2 of everything and he had this small vessel to do it in.

Now, if you take a dna sample of every animal, make it male and female, and then take off into a spaceship, you need a crew, and he had one, his children and their wives.

So this team of scientists took us from the first earth, and moved us to the second earth, and set up the cloneing, and then they started it all over

Thats why the bible and science conflict, cause we didn't start here, so the plantet is older. after all the stars were done before the earth was.


wait a minute, my own words spun a new theory

That who, Christ is coming again. Christ was a scout sent by Generals of Destruction aka G.o.D. His mission was to find the missing N.O.A.H. project. They artifically impregnated Mary, and thus he was born, but with advanced knowledge because of genetacally altered dna cloning.

When the people found out, they tried to get him. Though he led a revolt and when they did "kill him" he merely used it as a ruse to regenerate, and leave before they had a chance to find out. And he left the Holy Ghost aka a spy network, to keep an eye on us until he comes back with "all the saints" behind him and he strikes down all that oppose him and take us all to the "new Earth" or the one that they picked out instead of this one.
La Terra di Liberta
05-01-2005, 19:26
So you do believe in pink elephants?

O.K. Billions of people. We've been through this. How many does it take?

The belief that the Earth was flat lasted for generations upon generations. For some reason it had to be proven otherwise. Why is that a better default?



There are likely thousands of species of creatures that have yet to be discovered, so I won't rule it out. But the flat earth theory is no longer "recognized" as legitimate and it came after religion but early man worshipped a God(s) in one form or another and that has continued throught to today. The fact that it has existed this long and that so many people believe in (Muslims, Jews, Christians, Hindus, etc) shows that there must be something that tugs at us in that way, and yes numbers do matter. If only 5 people believed that the moon was flat but the rest of us believe it is round, who is more likely to be correct? This is not always the case but many times it is.
GoatZone
05-01-2005, 19:27
Stay on topic people, unless invisible or transparent relates to God.
It is on topic.

Just as there is no evidence that pink elephants and/or invisible pink unicorns do not exist, similarly there is no evidence that God exists.

Some of us require evidence to believe something exists, others require only faith.

And there's the answer to the OP.

(edit - forgot to quote)
Pythagosaurus
05-01-2005, 19:34
There are likely thousands of species of creatures that have yet to be discovered, so I won't rule it out. But the flat earth theory is no longer "recognized" as legitimate and it came after religion but early man worshipped a God(s) in one form or another and that has continued throught to today. The fact that it has existed this long and that so many people believe in (Muslims, Jews, Christians, Hindus, etc) shows that there must be something that tugs at us in that way, and yes numbers do matter. If only 5 people believed that the moon was flat but the rest of us believe it is round, who is more likely to be correct? This is not always the case but many times it is.
Yes, the flat earth theory was abandoned because it was proven false. The theory of God cannot possibly be proven false, so there has to be a better way to abandon it if it isn't true. Otherwise, there's no logical way to deduce that it's correct based on everybody else's beliefs. Now, how do you propose that we could test the verity of the theory of God?

How many people? I want a number.
UpwardThrust
05-01-2005, 19:36
Yes, the flat earth theory was abandoned because it was proven false. The theory of God cannot possibly be proven false, so there has to be a better way to abandon it if it isn't true. Otherwise, there's no logical way to deduce that it's correct based on everybody else's beliefs. Now, how do you propose that we could test the verity of the theory of God?

How many people? I want a number.
No but most of the “works” done by “him” can be proven false … which does not prove god false but can prove most of the founding of religions were false (and to some extent disproving them)
Pythagosaurus
05-01-2005, 19:37
It is on topic.

Just as there is no evidence that pink elephants and/or invisible pink unicorns do not exist, similarly there is no evidence that God exists.

Some of us require evidence to believe something exists, others require only faith.

And there's the answer to the OP.

(edit - forgot to quote)
And that's the real point. Why do believers require evidence for pink elephants and not for God? Is the only thing that makes God better than the flat earth theory the fact that it can't be disproven? It seems reasonable to me.
Pythagosaurus
05-01-2005, 19:40
No but most of the “works” done by “him” can be proven false … which does not prove god false but can prove most of the founding of religions were false (and to some extent disproving them)
We can't prove them false, though. Nobody can agree on when they happened. Nobody can agree that they happened. Nobody can claim that the bible (or whatever other record might be quoted as documentation) is genuine.
Grave_n_idle
05-01-2005, 19:53
So you admit its a choice then? Or is it based on experiences in our lives that shape how we see it?

Not at all.

I strongly argue against the concept that it's a choice.

Perhaps you would like to go and re-read it in that context, since I think you missed my point.

It must be one of two things: 1) Something fundamentally about 'ME' that makes me see something in that blot... or 2) Something fundamentally about my experiences, that makes me identify a certain shape in that blot.

And, since Rorshache tests give fairly reliable guides to the predominant concerns of a mind, it seems most likely that the 'experience' of reality influences our perceptions.

I didn't 'chose' not to see god. I look at the world (our inkblot) and I see a godless world. You look at the same blots, and see a mysterious creator.

Neither of us choses... and to IMPLY that I CHOSE to see a godless world, you are implying that my view of the world is somehow inferior to yours.
UpwardThrust
05-01-2005, 19:57
We can't prove them false, though. Nobody can agree on when they happened. Nobody can agree that they happened. Nobody can claim that the bible (or whatever other record might be quoted as documentation) is genuine.
Sure we can prove specific claims false.

For example the Christian faith bases their religion on the bible… we can disprove if a specific event happened … or a series.

We can disprove claims that pi is 3 and that it is not possible to fit all the worlds animals into the ship


Eventually you disprove the book the religion is based in


That does not effect the possibility that there is still a deity but we can disprove the religion
(I cant explain it clearer then this … maybe gravy can)
La Terra di Liberta
05-01-2005, 19:58
Not at all.

I strongly argue against the concept that it's a choice.

Perhaps you would like to go and re-read it in that context, since I think you missed my point.

It must be one of two things: 1) Something fundamentally about 'ME' that makes me see something in that blot... or 2) Something fundamentally about my experiences, that makes me identify a certain shape in that blot.

And, since Rorshache tests give fairly reliable guides to the predominant concerns of a mind, it seems most likely that the 'experience' of reality influences our perceptions.

I didn't 'chose' not to see god. I look at the world (our inkblot) and I see a godless world. You look at the same blots, and see a mysterious creator.

Neither of us choses... and to IMPLY that I CHOSE to see a godless world, you are implying that my view of the world is somehow inferior to yours.




No, I implied I chose to see a world with God. I didn't mean to say your view was inferior and I apologise if I came across like that. Bad wording.
Grave_n_idle
05-01-2005, 20:00
We can't prove them false, though. Nobody can agree on when they happened. Nobody can agree that they happened. Nobody can claim that the bible (or whatever other record might be quoted as documentation) is genuine.

I don't know... this seems like semantics.

If the ONLY evidence that leprachauns exist is "The Big Book of Leprachauns", and the first chapter says "All leprachauns are fish" and chapter two says "All leprachauns are toasters"...

We either have to fudge some reasoning that allows a fish to be a toaster, and both of them to be 'Leprachauns"...

or: we have to accept that there is NO evidence to support the existence of Leprachauns.

And, if nobody believed in Leprachauns BEFORE the book was written, the fact that ALL the evidence available is conflicted COULD be taken as 'proof' that the original Leprachaun-free vision of the world is more accurate.
La Terra di Liberta
05-01-2005, 20:03
Yes, the flat earth theory was abandoned because it was proven false. The theory of God cannot possibly be proven false, so there has to be a better way to abandon it if it isn't true. Otherwise, there's no logical way to deduce that it's correct based on everybody else's beliefs. Now, how do you propose that we could test the verity of the theory of God?

How many people? I want a number.



Here are the numbers from 2001:


Christianity-2 billion
Islam-1.3 billion
Hinduism-900 million
Sikhism-23 million
Judaism-14 million


This is from www.adherents.com I believe.
Grave_n_idle
05-01-2005, 20:04
No, I implied I chose to see a world with God. I didn't mean to say your view was inferior and I apologise if I came across like that. Bad wording.

But do you CHOOSE to see that world?

Do you not believe? Is your belief not a result of what you have seen, and experienced?

I don't think you CHOSE to find god, any more than I CHOSE to find 'no-god' ... and I think it is that word ('choosing') that causes the friction here.

But, if we are hitting the same wavelenght, then there is no foul.

Basically - I think - the Atheist believes in no gods, for the same reason YOU believe in god(s)... that is - the sum of your experiences proclaims it to be true, for you.
UpwardThrust
05-01-2005, 20:07
I don't know... this seems like semantics.

If the ONLY evidence that leprachauns exist is "The Big Book of Leprachauns", and the first chapter says "All leprachauns are fish" and chapter two says "All leprachauns are toasters"...

We either have to fudge some reasoning that allows a fish to be a toaster, and both of them to be 'Leprachauns"...

or: we have to accept that there is NO evidence to support the existence of Leprachauns.

And, if nobody believed in Leprachauns BEFORE the book was written, the fact that ALL the evidence available is conflicted COULD be taken as 'proof' that the original Leprachaun-free vision of the world is more accurate.


There ... almost what I ment to say :) (though I did not cary it as far) In this situation I went as far as disproving the book of leperchauns ... and therefore the religion that leperchauns is based on

Does not mean there is no leperchanuns ... just that the religion of the leperchauns in their current state is false
La Terra di Liberta
05-01-2005, 20:10
But do you CHOOSE to see that world?

Do you not believe? Is your belief not a result of what you have seen, and experienced?

I don't think you CHOSE to find god, any more than I CHOSE to find 'no-god' ... and I think it is that word ('choosing') that causes the friction here.

But, if we are hitting the same wavelenght, then there is no foul.

Basically - I think - the Atheist believes in no gods, for the same reason YOU believe in god(s)... that is - the sum of your experiences proclaims it to be true, for you.



Experiences allow people to see the same thing differently and that would stay true for God. Should I re-word the thread to "What has lead you to believe there is no God?"
Nasopotomia
05-01-2005, 20:17
Here are the numbers from 2001:


Christianity-2 billion
Islam-1.3 billion
Hinduism-900 million
Sikhism-23 million
Judaism-14 million


This is from www.adherents.com I believe.


Remember that many people who consider themselves 'Christian' aren't really. The often just believe vaguely in the concept of God.

Also remember that Judaism will include everyone of Jewish decent through the female line, regardless of how faithful they are, and that Hinduism is a collection of loosely-related faiths rather than a single solid one.
La Terra di Liberta
05-01-2005, 20:21
Remember that many people who consider themselves 'Christian' aren't really. The often just believe vaguely in the concept of God.

Also remember that Judaism will include everyone of Jewish decent through the female line, regardless of how faithful they are, and that Hinduism is a collection of loosely-related faiths rather than a single solid one.



Keep in mind, many those people who said they were Christians would believe there is a God, even if they don't practice the faith regularly.
Nasopotomia
05-01-2005, 20:21
We can't prove them false, though. Nobody can agree on when they happened. Nobody can agree that they happened. Nobody can claim that the bible (or whatever other record might be quoted as documentation) is genuine.

The problem with this is in the nature of proof, more than anything else. You prove things true. That's inherent in the nature of the word proof. For example, when people say:

"There's no proof he doesn't exist"

it's essentially tautologous. If there was proof, then he MUST exist to leave it there. You'll never find proof something ISN'T. Think about it.

Going to the Leprechauns, there's no proof that there aren't any. But there aren't. People are happy with that. Why should God get away with it when the Leprechaun's don't?
Omnibenevolent Discord
05-01-2005, 20:23
It's a question that has puzzled me because when I talk to various people that don't, they seem to have fairly different reasons, so you can't connect the dots. I'm simply curious why because believing or not believing in God is a concious choice we all make unless you're agnostic, so why folks?
Because believing in Eris was more fun.
La Terra di Liberta
05-01-2005, 20:25
The problem with this is in the nature of proof, more than anything else. You prove things true. That's inherent in the nature of the word proof. For example, when people say:

"There's no proof he doesn't exist"

it's essentially tautologous. If there was proof, then he MUST exist to leave it there. You'll never find proof something ISN'T. Think about it.

Going to the Leprechauns, there's no proof that there aren't any. But there aren't. People are happy with that. Why should God get away with it when the Leprechaun's don't?



The belief a God(s) exist is more plausable then little green men that live in Ireland and have a pot of gold at the end of the rainbow.
Nasopotomia
05-01-2005, 20:25
Keep in mind, many those people who said they were Christians would believe there is a God, even if they don't practice the faith regularly.

I did. But you can't really consider them Christian just because of a general belief that there is SOMETHING there. Islam has a God, but I don't think Osama bin Laden would think of himself as Christian. It's a bit like agnostics. They aren't sure, but they're seperated.

Most English people are either Agnostic, Atheist or have a sort of vague impression there's a God, but they'd say they were Christians. If you ask them about it in any depth, they'd probably say 'Yeah, but I thought if I died and He was there, I'd be better off believing.'. That's not religious really. But they're counted as Christians.
Drunk commies
05-01-2005, 20:25
The belief a God(s) exist is more plausable then little green men that live in Ireland and have a pot of gold at the end of the rainbow.
Why?
Willamena
05-01-2005, 20:26
I didn't know asking someone why they believe something was arrogant. Anyway, I was going to us the example of countries but then I thought people might say the scientific proof is there, so I swayed to something a little less clear. I don't believe in pink elephants because I never heard of anyone whos found one or anyone who believes they exist. If I went to my city zoo and one of the biologists or whatever they are called there if they do, I doubt I'd get a yes. I can see you saying the same thing about God but I'll wait just to make sure before I start yo defend it.
Is it safe to say that things can only believably exist if some human somewhere has found one? And that people have "found god"?
Omnibenevolent Discord
05-01-2005, 20:27
The belief a God(s) exist is more plausable then little green men that live in Ireland and have a pot of gold at the end of the rainbow.
But less plausable than the idea that both were made up by the minds of man...
La Terra di Liberta
05-01-2005, 20:31
And you people are the same ones accusing me of being arrogant. Holy shit, it's all "Why do believe in god, it's so weird, no proof, blah blah blah". Why the hell does it matter if I believe in him and not in pink elephants and leprechauns? Honestly, if you give a damn, don't. If people have to constantly defend their faith and then when they possibly ask someone else why they believe what they do and then get the shit ridiculed out of them, then thats just wrong.
La Terra di Liberta
05-01-2005, 20:32
Is it safe to say that things can only believably exist if some human somewhere has found one? And that people have "found god"?


I changed that view in a later post allowing for it to exist.
Nasopotomia
05-01-2005, 20:32
But less plausable than the idea that both were made up by the minds of man...

EXACTLY. Man's been making stuff up for 40,000 years. Why is God somehow immune to this, when we've decided so much other stuff is false now?


It's not even like God is the first thing we invented. In the form you see him today, hes less than 6,000 years old. Leprechaun's aren't that much younger.
La Terra di Liberta
05-01-2005, 20:37
I'll leave it at this: I believe in God, most of the rest of the people on this thread don't. Fine. I have my reasons, you have yours. Fine. I was simply curious why people didn't but some how, it turned into why I did.
Grave_n_idle
05-01-2005, 20:42
Experiences allow people to see the same thing differently and that would stay true for God. Should I re-word the thread to "What has lead you to believe there is no God?"

That would have been a much better question - and would have avoided half of the trouble, I suspect.
Nasopotomia
05-01-2005, 20:42
I'll leave it at this: I believe in God, most of the rest of the people on this thread don't. Fine. I have my reasons, you have yours. Fine. I was simply curious why people didn't but some how, it turned into why I did.

No no no no no no. We're asking the questions that religion can't answer, because these are the questions we asked ourselves before we stopped believing. It's not an attack on you. Believe what the hell you like, we almost certainly never going to meet face to face and I don't care what you think'll happen to you when you die.

The point is, as God is no more plausible than the leprechauns and the pink elephants, we don't believe in him any more than those things. That's the answer to your question. There you go. Happy now? Or do you want to claim we're attacking you and your religion?
GoatZone
05-01-2005, 20:45
What Nasopotomia said.
Grave_n_idle
05-01-2005, 20:47
I'll leave it at this: I believe in God, most of the rest of the people on this thread don't. Fine. I have my reasons, you have yours. Fine. I was simply curious why people didn't but some how, it turned into why I did.

That's because, it is essentially the same thing.

You believe in your 'god', because it seems right to you - it just 'works' somehow, with what life has given you, and what you have learned.

I don't believe in your 'god', because it doesn't seem right to me - it just 'doesn't work', somehow, with what life has given me, and what I have learned.

It really is that simple.
Slinao
05-01-2005, 20:56
Science without religion is lame, religion without science is blind.
Nihilistic Beginners
05-01-2005, 21:20
I'll leave it at this: I believe in God, most of the rest of the people on this thread don't. Fine. I have my reasons, you have yours. Fine. I was simply curious why people didn't but some how, it turned into why I did.

Why don't you beleive in Buddha?
Drunk commies
05-01-2005, 22:05
Science without religion is lame, religion without science is blind.
How do you figure that science without religion is lame? What does one have to do with the other?
Uldaedia
05-01-2005, 22:13
God is an imaginary freind for adults. One of my favorite quotes.

I used to be a die-hard mormon. But there are just to many hypocritical staements for comfort.
1) Magic doesn't exist. (Hmmm....what do you call a big man in the sky that snaps his fingers and makes planets?)
2)Gohsts don't exist. (what do you call an angel?)
3)God loves you but if you screw up you're going to hell (That one speaks for itself)
4) God made Adam and Eve. Then they bred with each other and had their children breed with each other and thatls how we all got here. (If god can make people why didn't he make enough so that brothers and sisters didn't have to marry?)
5) And the all-mighty asnwer for all of this is....."YOU MUST HAVE FAITH!*

I must have faith? Faith in a guy who forced siblings to have sex with each other? Faith in a guy who would send his children to hell? Faith in someone who, according to the bible, murdered thousands of people bacause they made a mistake? Faith in someone who leades his "followers" to believe that anyone who doesn't believe in him is an advocate for the devil? Faith in someone who lets war and famine and desease take place on his planet? I think not!

There are just too many loopholes. Yeah, Adam and Eve, whatever. Did he create the cavemen, too? Oh, no, sorry, no questions! Gotta have faith! Right...

I live in a democracy. I guess I just don't fancy the idea of serving no one but an all-mighty guy in the sky who I've never seen before for the rest of my life, and attending churches that won't let females become preists, bishops, etc. (Why? Oh, because they don't look like god. He's a guy you know. Can't have women in conrtol. Better leave them to make dinner. But god loves everybody!) Not my idea of a life.

By the way, I'm an agnostic pagan. *g*
Uldaedia
05-01-2005, 22:17
I relized after I posted that that it sounded kind of attackish. It wasn't meant to be. Those were the questions I asked myself, and ask myself every day. Sorry if it sounded mean.
Maledicti
05-01-2005, 22:32
I don't know if anyone has posted this or anything similar, but....


"I contend we are both atheists, I just believe in one fewer god than you do. When you understand why you dismiss all the other possible gods, you will understand why I dismiss yours."

-Stephen Roberts

EDIT: Yes, someone already posted it. My apologies for not reading all the posts and needlessly reposting a quote.
UpwardThrust
05-01-2005, 22:49
I don't know if anyone has posted this or anything similar, but....


"I contend we are both atheists, I just believe in one fewer god than you do. When you understand why you dismiss all the other possible gods, you will understand why I dismiss yours."

-Stephen Roberts
Another good quote :) I got to write these down
Willamena
05-01-2005, 22:55
How do you figure that science without religion is lame? What does one have to do with the other?
"...representatives of science have often made an attempt to arrive at fundamental judgments with respect to values and ends on the basis of scientific method, and in this way have set themselves in opposition to religion. These conflicts have all sprung from fatal errors. Now, even though the realms of religion and science in themselves are clearly marked off from each other, nevertheless there exist between them two strong reciprocal relationships and dependencies. Though religion may be that which determines the goal, it has, nevertheless, learned from science, in the broadest sense, what means will contribute to the attainment of the goals it has set up. But science can only be created by those who are thoroughly imbued with the aspiration toward truth and understanding. This source of feeling, however, springs from the sphere of religion. To this there also belongs the faith in the possibility that the regulations valid for the world of existence are rational, that is, comprehensible to reason. I cannot conceive of a genuine scientist without that profound faith. The situation may be expressed by an image: science without religion is lame, religion without science is blind..."

There is more on the link (http://condor.stcloudstate.edu/~lesikar/einstein/Einstein2b.html).
Drunk commies
05-01-2005, 23:12
"...representatives of science have often made an attempt to arrive at fundamental judgments with respect to values and ends on the basis of scientific method, and in this way have set themselves in opposition to religion. These conflicts have all sprung from fatal errors. Now, even though the realms of religion and science in themselves are clearly marked off from each other, nevertheless there exist between them two strong reciprocal relationships and dependencies. Though religion may be that which determines the goal, it has, nevertheless, learned from science, in the broadest sense, what means will contribute to the attainment of the goals it has set up. But science can only be created by those who are thoroughly imbued with the aspiration toward truth and understanding. This source of feeling, however, springs from the sphere of religion. To this there also belongs the faith in the possibility that the regulations valid for the world of existence are rational, that is, comprehensible to reason. I cannot conceive of a genuine scientist without that profound faith. The situation may be expressed by an image: science without religion is lame, religion without science is blind..."

There is more on the link (http://condor.stcloudstate.edu/~lesikar/einstein/Einstein2b.html).
I disagree completely. "representatives of science have often made an attempt to arrive at fundamental judgements with respect to values" Bull. Science isn't concerned with values. Only with learning about the physical universe.

"Science can only be created by those who are thoroughly imbued with the aspiration toward truth and understanding. The source of feeling, however, springs from the sphere of religion."

Correct me if I'm wrong, but I take this to mean that religion is necessary to inspire the hunger for truth and understanding. I don't find this to be true at all. I'm not a religious person, yet even as a layman I constantly pursue scientific knowledge.
Willamena
05-01-2005, 23:40
I disagree completely. "representatives of science have often made an attempt to arrive at fundamental judgements with respect to values" Bull. Science isn't concerned with values. Only with learning about the physical universe.
Not sure what he was getting at there, but perhaps it was a topic for heated discussion in his day. Personally, I fell asleep half-way through it. ;-)

"Science can only be created by those who are thoroughly imbued with the aspiration toward truth and understanding. The source of feeling, however, springs from the sphere of religion."

Correct me if I'm wrong, but I take this to mean that religion is necessary to inspire the hunger for truth and understanding. I don't find this to be true at all. I'm not a religious person, yet even as a layman I constantly pursue scientific knowledge.
That's how I read it, though of course not talking about participation in organized religion, but the religiously enlightened person participating in truth. From the link:
"...a person who is religiously enlightened appears to me to be one who has, to the best of his ability, liberated himself from the fetters of his selfish desires and is preoccupied with thoughts, feelings, and aspirations to which he clings because of their superpersonal value. It seems to me that what is important is the force of this superpersonal content and the depth of the conviction concerning its overpowering meaningfulness, regardless of whether any attempt is made to unite this content with a divine Being, for otherwise it would not be possible to count Buddha and Spinoza as religious personalities."

I think he is saying that the religiously inclined person has put himself in a position of being subordinate to greater forces, like those that control the universe, and in this humbled state is in a position to pursue research for research's sake rather than for personal gain (either financial or notorious).
The White Hats
06-01-2005, 00:23
Not sure what he was getting at there, but perhaps it was a topic for heated discussion in his day. Personally, I fell asleep half-way through it. ;-)

...

He could well have been referring to philisophical approaches to morality such as utilitarianism or logical positivism. The former using reason to set rational standards and the latter insisting on verifiability. In terms of relevance, utilitarianism is the basis for ethics and morality in economics, which (IIRC) was going through a major development in the thirties and forties. Similarly with sociology and political science.

So his 'representatives of science' may have been applying the scientific method to ethics rather than actually being traditional (physical/natural) scientists. In other words, social scientists.
Zachnia
06-01-2005, 00:54
People want proof nowadays, simple as that.

But then again we don't have proof on gravity... does that mean gravity doesnt exist? we dont' have proof that something makes up an electron (do we?), yet people believe there to be so.



If you drop something, then that's proof that gravity exists. All gravity is is a name for something we knew already existed.

of course something makes up an electron, because we can sense it.

I basically don't believe in God because there's no proof. And I don't see any reason to believe in it.
East Coast Federation
06-01-2005, 01:03
It's simple as this.
As people become smarter, and as humans become more aware of where we really came from there is no need for a god or some kind of higher power.

Even as recenetly as the Turn of the centuray few could explain why or how we are here.
But snice reseach and people like Darwin have showed people a theroy that is probably correct. The need for a higher power is lost.
Boonytopia
06-01-2005, 01:04
Maybe this Stephen Roberts quote will help you understand why some people do not believe in God -

"I contend that we are both atheists. I just believe in one fewer god than you do. When you understand why you dismiss all the other possible gods, you will understand why I dismiss yours."

That's an excellent quote. Who is Stephen Roberts? I've never heard of him before.
Meaning
06-01-2005, 01:17
it just faith, we don't have it b/c we been forsaken or we never needed to have it. so we have no faith, so we don't belive
Boonytopia
06-01-2005, 01:18
You mention how a question is worded to sound arrogant but anyway, the belief in God is more believable because it has exist since very early in the history of man and has continued down throught the generations. If it were such a weak theory, it wouldn't have last very long but it has. Maybe pink elephants live at the bottom of the sea, although give the current elephants live on land, thats highly unlikely or maybe an small island in the South Pacific , Asia or Africa. that has not yet been visited. You can give a bit on either one.

Just because a belief has existed for a long time, or a great number of people believe in it, doesn't make it right. The flat earth idea is a very good example of this.
Meaning
06-01-2005, 01:27
Just because a belief has existed for a long time, or a great number of people believe in it, doesn't make it right. The flat earth idea is a very good example of this.


ur right. if a great number of people believe in something and it was right then everyone would be Muslims just b/c most of the world's population is Muslim, but no
Industrial Experiment
06-01-2005, 01:30
I suppose I don't simply because I never did. It was never really forced on, though I did attend Church and read a Children's Bible, I never actually believed. Going to mass was just something I did, not some homage to an omnipotent being. I saw the Children's Bible as just another book (I was a very avid reader).

I guess it just never occured to me and atheism became my default state.
Nihilistic Beginners
06-01-2005, 01:38
I suppose I don't simply because I never did. It was never really forced on, though I did attend Church and read a Children's Bible, I never actually believed. Going to mass was just something I did, not some homage to an omnipotent being. I saw the Children's Bible as just another book (I was a very avid reader).

I guess it just never occured to me and atheism became my default state.

You weren't predisposed to believe, even though you probably culturally programmed to believe.
Compuq
06-01-2005, 01:43
Its makes sense to me that there is no god.

(Example) Remember when you were a kid and believed in the tooth fairy, Easter bunny and Santa clause.

Looking back, what is more logical?

A fairy that travels the world looking for children’s teeth under their pillows and replacing it with money......... or your parents took the teeth and put money there.

A magic giant bunny the travails the world (In one night) giving out chocolate eggs...... or parents/guardians put the chocolate there.

or a large fat man that lives at the north pole with his elves making toys all year and then delivers them to children(in one night) He knows if you been bad or good......or parents/guardians buy presents for there children

Applying that logic

An omnipresent God, he lives in a paradise. It always is watching you and knows what you are thinking. If you do bad things in your life you will go to hell. Where you will live in Miserere for eternity...but he "loves" you. He has always existed and always will……..or none of this is true and was an something people made up a long time ago to explain things they did not understand, to add purpose to their lives or propagated by the church to keep order (or stay in power) in primitive societies.
Meaning
06-01-2005, 01:55
....
or a large fat man that lives at the north pole with his elves making toys all year and then delivers them to children(in one night) He knows if you been bad or good......or parents/guardians buy presents for there children
.....
.


first there was saint nick who did this (or at least gave gift to good kids). so these could happen. he could deliver the the toys b/c time differents and only some religions belive in him. and good or bad u still get something. so i know its not true and shit but there could be a hair chance that, that could happen. it's plausable.
Erehwon Forest
06-01-2005, 02:17
first there was saint nick who did this (or at least gave gift to good kids). so these could happen. he could deliver the the toys b/c time differents and only some religions belive in him. and good or bad u still get something. so i know its not true and shit but there could be a hair chance that, that could happen. it's plausable.Yeah, uh, if you go with the assumption that (nearly) omnipotent critters are plentiful in the multiverse, then Santa Claus (and similar fat jolly winter solstice gift givers) are plausible. It would still require some major miracle-work, in particular the nasty stretching of time.

And which religions believe in Santa Claus, exactly? As far as I knew, it was a largely non-religious thing (seeing as how the majority of kids who believe in Santa don't have a fucking clue what "religion" or "god" mean), although it is culturally related to some religions. In Finland, for example, Santa is completely irreligious.
Slinao
06-01-2005, 02:23
How do you figure that science without religion is lame? What does one have to do with the other?
I'm just quoteing Einstien.
Zeppistan
06-01-2005, 03:01
So, I asked myself why I should believe in a deity who started populating the world via two nudists who took dietary advice from a talking serpent, who showed many instances of cruelty to the creations he was supposed to love, and who - along with another of his creations - was willing to allow those he loved to suffer an eternity of torture simply to satisfy the answer to a wager as to whether they would choose his path or not when he gives little indication of his existance to help us do so.

And I told myself that the stories make no sense and contradict themselves between his actions and his message. If there were a supreme being, then when he cast us in his image he did so including the worst parts of us.

Ergo, I found it impossible to believe that he were infallible, which contradicted the central tenent of what (s)he is supposed to be.

Given this logical contradiction, I had to conclude that God - as described in the bible- does not exist.
Markdorf
06-01-2005, 03:15
Well, I do not really believe in god, but... there is no evidence to prove or dissprove its existance. So I can not make a solid arguement either way. I would have liked to believe in something, because the fact that death is just the end... truthfully scares me. This might sound strange, but I always think things like "What does it matter if you die, or when you die, or how you die? Because no matter what you will still die, and you will no longer exist. Even the most kind people have horrible things happen to them, and the most evil people have wonderful lives. And no matter what, no matter how hard you try, the same punishment awaits all of us, death." Truthfully, I would rather spend an eternity in hell being in pain then to not exist. But I still can not make my self believe. I think it has something to do with the fact that most of the time the religions are all saying they are right, and that all the others are wrong. It might have been a little easier for me if I had been around in the greco-roman times because at least their religions were more open minded, and accepted each other for the most part, because being polytheistic they are usually more tolerant of other religions. I might have been able to accept faith then. Although, it would never be something that would control my life, and would not let it change any already existing opinions I had before I accepted the religion once I was old enough to understand religion.
Meaning
06-01-2005, 03:23
Yeah, uh, if you go with the assumption that (nearly) omnipotent critters are plentiful in the multiverse, then Santa Claus (and similar fat jolly winter solstice gift givers) are plausible. It would still require some major miracle-work, in particular the nasty stretching of time.

And which religions believe in Santa Claus, exactly? As far as I knew, it was a largely non-religious thing (seeing as how the majority of kids who believe in Santa don't have a fucking clue what "religion" or "god" mean), although it is culturally related to some religions. In Finland, for example, Santa is completely irreligious.


i thought i was christains b/c jewish people don't celebrate christmas b/c of christ but they don't belive in santa either
Erehwon Forest
06-01-2005, 03:43
The majority of the people who believe in Santa Claus probably live with parents who have christian values, which is certainly not the same thing as christianity believing in Santa Claus, whatever that would mean.

Still, if you want to argue that believing in a god/gods somehow makes sense, you should probably refrain from comparing god(s) to Santa Claus.
Grave_n_idle
06-01-2005, 03:44
first there was saint nick who did this (or at least gave gift to good kids). so these could happen. he could deliver the the toys b/c time differents and only some religions belive in him. and good or bad u still get something. so i know its not true and shit but there could be a hair chance that, that could happen. it's plausable.

Except that the basis of the story is a real person who lived about 1700 years ago - which means, being realistic, that he must have been dead for ... say, 1600 years...

Plausible? Not so much...
Meaning
06-01-2005, 03:46
Except that the basis of the story is a real person who lived about 1700 years ago - which means, being realistic, that he must have been dead for ... say, 1600 years...

Plausible? Not so much...


i'm sure he had sons.............. and dude i just brought it up to joke around and get some heat off from the guy who oringaly posted the thread
Siesatia
06-01-2005, 04:02
Why don't I believe in your god(s)? Because religion is the big school club or team that everyone believes they have to be in to be cool. There are no reasons for religion anymore, since all it was used for, is to explain things that we have found a real reason for.
Another reason? Because he doesnt do anything if he does excist. I mean, if he doesnt want us to screw up our genetic code... Why doesnt he send down a sign that it is bad? Or hows this, why did he kill the dinosaurs? I have asked this question to many christian friends... their answer... Because he got bored with them. Well, maybe he will get bored with us. Maybe he is in actuallity a big baby, who breaks his toys when they get old?
In my opinion, humans are a plauge of the earth, and even the universe in general, since we are already polluting the other planets with spent probes, and vehicles. So, if he could forsee this, being omnipitent and all, why did he not create a more sypathetic race, or at least smarter?
The church believes it can explain everything away as 'gods creation' but science will prove them wrong in the end. Eventually, religion will fall to a new wave of scientific thought. Another revolution, that will drastically change the human perspective on life, and the universe. That is what history shows, and as we all know, history tends to repeat itself. I just wish it would be in my lifetime.
Stabbatha
06-01-2005, 04:05
I don't know whether I believe or not, but I simply feel that when I'm dead we'll see who's right...kinda like in the South Park movie :D

PS: God is a buddist just so you know ;) Jk of course but yea. I do not believe but I don't not believe. I'm Captain Undecided!
La Terra di Liberta
06-01-2005, 04:08
I don't know whether I believe or not, but I simply feel that when I'm dead we'll see who's right...kinda like in the South Park movie :D

PS: God is a buddist just so you know ;) Jk of course but yea. I do not believe but I don't not believe. I'm Captain Undecided!


You're agnostic then. And yes everyone, I came back after my tyrade. Went over to a friends house and went on PS 2 for a while, had supper and have since cooled off.
International Terrans
06-01-2005, 04:49
If half these people took a philosophy course, they'd see just how "logical" their "arguments" really are.

Quite simply, believing in God is better for you. Just like Guiness. Ask Pascal when you die. I'm sure he'd be more than willing.

Drink a pint of religion, buds. I'm sure if churches offered free Guiness after every service or Mass, there'd be alot more converts.

The proof of God is not tangible at all. The proof of His existence is in our own hearts. While many people were brought up to believe in God (as I was), their hearts are not in it. I was one of these people - until, for the first time in my life, I actually truly prayed (it was a bad time for me) - and my prayer was heard. It was freaky. I get home from Mass and it happened. Most eerie thing in my entire life.

After that, I've been convinced. The same just needs to happen with everybody else - make a leap of faith. Many people don't bother. That's their choice. I respect it. I only wish that all the atheists and agnostics would do the same to me - even if I like to refer to agnostics as the concientious objectors in the wars of religion ;)
Bill Mutz
06-01-2005, 04:56
It's a question that has puzzled me because when I talk to various people that don't, they seem to have fairly different reasons, so you can't connect the dots. I'm simply curious why because believing or not believing in God is a concious choice we all make unless you're agnostic, so why folks?I refuse to believe in a god who insists upon being admired all the time. Besides I find the religion depressing.
Gnostikos
06-01-2005, 05:01
If half these people took a philosophy course, they'd see just how "logical" their "arguments" really are.
You haven't read much of the agnostic or atheist philosophy, have you? I highly reccomend Jean-Paul Sartre, he's fecking incredible.

The same just needs to happen with everybody else - make a leap of faith. Many people don't bother.
It's not that we don't bother, it's that we think that it's foolhardy to do so. Making leaps of faith seem to be begging for disaster in my eyes.
La Terra di Liberta
06-01-2005, 05:09
I refuse to believe in a god who insists upon being admired all the time. Besides I find the religion depressing.


If you mean Christianity, I never said that outright. There are two other large monothestic religions, and they share the same God, just under a different name. So do all 3 depress you then.
International Terrans
06-01-2005, 05:09
You haven't read much of the agnostic or atheist philosophy, have you? I highly reccomend Jean-Paul Sartre, he's fecking incredible.
Most of the stuff I read was Nietzche. It disgusted me. Most of what I've read in general has been ill-informed and arrogant atheists ranting on about the problem of evil. However, when I see a well-informed one, I do give them kudos. However, how much Aquinas or Maritain or Stein have you read? The same problem goes both ways.

It's not that we don't bother, it's that we think that it's foolhardy to do so. Making leaps of faith seem to be begging for disaster in my eyes.
Oh really? My philosophy teacher on Monday morning quoted someone, a French philosopher (I forget his name) from either the late 19th or early 20th century, who said that the only real philosophical question was whether or not to commit suicide.

If you answer, "no", then you assume that life has meaning. This leads, not always but often, to God. He also said that the Universe may be absurd, like Satre (or was it Camus...?) said, but it is better for us to go about as if it had meaning regardless.

Assuming that the Universe has meaning of any sort, anywhere, anyhow, seems pretty close to a leap of faith to me.
Pythagosaurus
06-01-2005, 05:29
If half these people took a philosophy course, they'd see just how "logical" their "arguments" really are.

Quite simply, believing in God is better for you. Just like Guiness. Ask Pascal when you die. I'm sure he'd be more than willing.

Drink a pint of religion, buds. I'm sure if churches offered free Guiness after every service or Mass, there'd be alot more converts.

The proof of God is not tangible at all. The proof of His existence is in our own hearts. While many people were brought up to believe in God (as I was), their hearts are not in it. I was one of these people - until, for the first time in my life, I actually truly prayed (it was a bad time for me) - and my prayer was heard. It was freaky. I get home from Mass and it happened. Most eerie thing in my entire life.

After that, I've been convinced. The same just needs to happen with everybody else - make a leap of faith. Many people don't bother. That's their choice. I respect it. I only wish that all the atheists and agnostics would do the same to me - even if I like to refer to agnostics as the concientious objectors in the wars of religion ;)
Actually, I've taken courses in formal logic in both the philosophy and mathematics departments at a first tier university. I aced every test with no effort whatsoever. I've also repeatedly ranked in the top 100 math students in the country, both in high school contests (ARML, USAMO) and in college (Putnam). Now, let's talk logic.

"The proof of His existence is in our own hearts." LOL

Everything after that is merely a testimonial. Thanks for the lesson.

You'll note that I've already made reference to Pascal's statements in my original post about pink elephants.

For example, I make the following proposition. A million people tell you that if you believe that there are naturally occurring pink elephants, then you will win the lottery. If you don't, your house will burn down. There's no particular disadvantage to believing in pink elephants. Thus, a simple cost-benefit analysis indicates that everybody should believe in pink elephants. Now, do you believe in pink elephants? I didn't think so. Why would you make such a choice? Do you want your house to burn down?

Pascal's argument is defeated by his own faulty axioms. If people had a conscious choice about whether or not to believe in God, then he would be correct. However, it would also lead people to believe in every religion simultaneously, or at least as many as possible without contradictions. Is that your intent?
Gnostikos
06-01-2005, 05:43
Most of the stuff I read was Nietzche. It disgusted me. Most of what I've read in general has been ill-informed and arrogant atheists ranting on about the problem of evil. However, when I see a well-informed one, I do give them kudos. However, how much Aquinas or Maritain or Stein have you read? The same problem goes both ways.
Very good point.

Oh really? My philosophy teacher on Monday morning quoted someone, a French philosopher (I forget his name) from either the late 19th or early 20th century, who said that the only real philosophical question was whether or not to commit suicide.

If you answer, "no", then you assume that life has meaning. This leads, not always but often, to God. He also said that the Universe may be absurd, like Satre (or was it Camus...?) said, but it is better for us to go about as if it had meaning regardless.
So...you're a Christian (I'm assuming from your earlier post, but you're at least one of the major monotheistic denominations), and you just gave such a deep and succinctly put argument? If I encountered more like you, perhaps I would read some of the philosophers I disagree with. I do of course believe that life has meaning, which is propagation of the species. Basic biological conept there. My religious and scientific views are inextricable--I am a physiolatrist.

I find more beauty in the natural cycles of nature then I've ever seen in any church or temple. I can sit and watch insects for hours, seeing them gather and feed, hunt and die. A single leaf fallen on the ground is so intoxicatingly stunning if you view it right. Its imperfection brought about by evolution, rendering it more perfect than Plato's mathematical ideals. Its tissues are being decomposed as I watch, by microorganisms beyond my sight, preparing to feed future generations. And the cycles of life, birth, death, and rebirth, will continue until the cessation of the planet, or at least its ability to sustain life (including extremophiles). And it will all vanish, just a passing phase in the whole cosmic scheme of the universe. A candle snuffed out before it even truly began. How glorious!

So I'm kind of on the fringe there, but my physiolatry (I'm also agnostic, by the by) seems to me to give more meaning to life and the universe than any concieved God could.
Uldaedia
06-01-2005, 05:48
Oh really? My philosophy teacher on Monday morning quoted someone, a French philosopher (I forget his name) from either the late 19th or early 20th century, who said that the only real philosophical question was whether or not to commit suicide.

If you answer, "no", then you assume that life has meaning. This leads, not always but often, to God. He also said that the Universe may be absurd, like Satre (or was it Camus...?) said, but it is better for us to go about as if it had meaning regardless.

Assuming that the Universe has meaning of any sort, anywhere, anyhow, seems pretty close to a leap of faith to me.

Sorry, but I have to say something to that. You may have seen my post before. I like to call myself an agnostic pagan. What you say up above shocked me. Just because we don't want to commit suicide means we must believe in a God? I'm sorry, but I'd like to know where you came up with that logic. Just because I have meaning in my life doesn't mean a big guy in the sky is the onw ho put it there!

Can't people have meaning because they know that people are gonna live on this planet after them, and they want to live and affect it for our ancestors? They want to keep it intact so that future generations can read about us in their history books and know that we accomplished something? Have we become such a group of followers that we need someone to tell us that life has a meaning?

I used to go to church. It wasn't healthy for me. I feel as if I live a much better and stronger life now. I have a better relationship with my family, with friends, have more fun, do exciting and adventurous things, and am not afraid to speak my mind. I, for one, answered "no", but not because I want to make it to "heaven". Not because I "love my father (god)". I have news: that guy that everybody keeps talking about is not my father. I answered "no" because I want to make a difference in the world I live in, no matter how that world came to be.
Willamena
06-01-2005, 05:51
A spaceman came traveling on his ship from afar.
'Twas light years of time since his mission did start.
And over a village he halted his craft,
And it hung in the sky like a star, just like a star.

He followed a light and came down to a shed.
Where a mother and child were lying there on a bed,
A bright light of silver shone round his head,
And he had the face of an angel, and they were afraid.

Then the stranger spoke, he said, "Do not fear,
I come from a planet a long way from here,
And I bring a message for mankind to hear,
And suddenly the sweetest music filled the air…

And it went La La La La La La La La La La…
Peace and goodwill to all men, and love for the child.

This lovely music went trembling through the ground,
And many were wakened on hearing that sound,
And travelers on the road, the village they found,
By the light of that ship in the sky, which shone all around.

And just before dawn at the paling of the sky,
The stranger returned and said, "Now I must fly,
When two thousand years of your time has gone by,
This song will begin once again, to a baby's cry…"

Oh the whole world is waiting, waiting to hear the song again,
There are thousands standing on the edge of the world,
And a star is moving somewhere, the time is nearly world,
This song will begin once again, to a baby's cry…
____________

What does this song make you feel?

That is myth.

That is what religion means.
Pythagosaurus
06-01-2005, 05:57
Sorry, but I have to say something to that. You may have seen my post before. I like to call myself an agnostic pagan. What you say up above shocked me. Just because we don't want to commit suicide means we must believe in a God? I'm sorry, but I'd like to know where you came up with that logic. Just because I have meaning in my life doesn't mean a big guy in the sky is the onw ho put it there!

Can't people have meaning because they know that people are gonna live on this planet after them, and they want to live and affect it for our ancestors? They want to keep it intact so that future generations can read about us in their history books and know that we accomplished something? Have we become such a group of followers that we need someone to tell us that life has a meaning?

I used to go to church. It wasn't healthy for me. I feel as if I live a much better and stronger life now. I have a better relationship with my family, with friends, have more fun, do exciting and adventurous things, and am not afraid to speak my mind. I, for one, answered "no", but not because I want to make it to "heaven". Not because I "love my father (god)". I have news: that guy that everybody keeps talking about is not my father. I answered "no" because I want to make a difference in the world I live in, no matter how that world came to be.
Indeed, we're programmed to want those things. That's how our species survived before it had all of its glorious advances.

Even if there's probably no benefit to living, there's certainly no benefit to dying (unless, of course, the largest group of religions that don't contradict each other encourage suicide). Thus, a simple cost-benefit analysis shows that you should not commit suicide.
Dakini
06-01-2005, 06:01
If half these people took a philosophy course, they'd see just how "logical" their "arguments" really are.

Quite simply, believing in God is better for you. Just like Guiness. Ask Pascal when you die. I'm sure he'd be more than willing.
i find it amusing that you chastise those with illogical arguments for not taking a philosophy course.

and then you make a refrence to pascal's wager... which is rediculously easliy debunked. maybe you shoudl take a philosophy class so you can learn to think and see the problems with such statements as pascal's wager.
Dakini
06-01-2005, 06:06
I can prove that there are no invisible pink unicorns. Pink means that they reflect that wavelength of light. Invisible means light goes right through them. Since no object could have both those properties, your invisible pink unicorns cannot exist.
ah, but they're only visible in the 5th dimension and they can enter into our three dimensions when they please, appearing pink in both places. however, when they are only observing our world, they are invisible.

duh.
Hialti
06-01-2005, 06:07
Though I believe in God, I also believe you should choose what is best for you, even if someone else thinks it's wrong.
International Terrans
06-01-2005, 06:24
Nice to see I created such a stir. Time to clear up some misconceptions.

First of all: I do realise that there are some atheists/agnostics with decent arguments, who have taken philosophy - just like there are some theists out there who've taken philosophy also, myself included. There are exceptions. But look back on this thread - just how many people have made coherent, sensible, well-argued arguments? A handful at best.

Even if there's probably no benefit to living, there's certainly no benefit to dying (unless, of course, the largest group of religions that don't contradict each other encourage suicide). Thus, a simple cost-benefit analysis shows that you should not commit suicide.
Even if there is probably no God, then there is certainly no downside to believing in Him (unless, of course, it is a whacked-out cult God). Thus, A simple cost benefit analysis shows that it is better to believe in God.

Hey, guess what that is? Pascal's Wager with different wording. As your post, in essence, just was. Wow. Now, no offence intended, stuff it.

"The proof of His existence is in our own hearts." LOL
I bet you'd say the same to someone who claimed they were in love. How do you know you are in love? You cannot. It just happens. The same is true with believe in God. Do not scoff at someone's experiences - if someone told me they didn't believe in God because all their relatives and friends just died, I'd respect that (provided they weren't feeding me bull). Show a tad of common courtesy - logic only proves that it is better to treat people better, because you get far more in return.

Sorry, but I have to say something to that. You may have seen my post before. I like to call myself an agnostic pagan. What you say up above shocked me. Just because we don't want to commit suicide means we must believe in a God? I'm sorry, but I'd like to know where you came up with that logic. Just because I have meaning in my life doesn't mean a big guy in the sky is the onw ho put it there!
Man, I'm not the one to argue with. Argue with that long-dead French philosopher, it's his quote, not mine. Besides, I said that it leads often to God, not always. You can't read my posts quickly, or you miss alot.

i find it amusing that you chastise those with illogical arguments for not taking a philosophy course.

and then you make a refrence to pascal's wager... which is rediculously easliy debunked. maybe you shoudl take a philosophy class so you can learn to think and see the problems with such statements as pascal's wager.
Hey, guess what? If you actually bothered to read my post, you would see the reference to "Monday morning" and "my philosophy teacher". I see the fallacies in Pascal's Wager, it's not why I believe in God. That was a joke - learn to see humour. Yeesh.

Maybe you should take an English class, for that poor spelling of yours? ;)

So...you're a Christian (I'm assuming from your earlier post, but you're at least one of the major monotheistic denominations), and you just gave such a deep and succinctly put argument? If I encountered more like you, perhaps I would read some of the philosophers I disagree with.
I pride myself on recieving compliments like that - which I've actually gotten several times on the NS forums, when so much is mere flame.

And yes, I am a Christian - a practising Roman Catholic, to be exact. Notice the three philosophers I suggested reading were all Catholics, hehe, even if one was a Jewish convert, another was a crazed Frenchman, and the last lived in the Middle Ages.

My philosophy teacher is a genius - he was raised as an atheist, and converted to Catholicism at the age of 21. He has a way of making things make sense, even if it does get a bit fuzzy at times. I think I'd probably be his protege.

I find more beauty in the natural cycles of nature then I've ever seen in any church or temple. I can sit and watch insects for hours, seeing them gather and feed, hunt and die. A single leaf fallen on the ground is so intoxicatingly stunning if you view it right. Its imperfection brought about by evolution, rendering it more perfect than Plato's mathematical ideals.
Actually, I tend to find God's presence much stronger in nature when I'm searching for Him than Mass when I am not (sometimes of one, sometimes of the other). I actually view that perfection as one of my own personal proofs of God - some of my best ideas have come from nature, and discussion with others therein.

Please, people, from now on, read thoroughly so I don't have to respond so much.
Uldaedia
06-01-2005, 07:52
Man, I'm not the one to argue with. Argue with that long-dead French philosopher, it's his quote, not mine. Besides, I said that it leads often to God, not always. You can't read my posts quickly, or you miss alot.

First of all, I am no man,(I hate it when people assume that), secondly, if you don't stand by the quote then why did you post it? I did read your post thourghly, and I saw the "often", and I understood it too. What got me was the quote itself. I'm "arguing" with you because you posted it and didn't say anything against it, so that led me to believe that you agreed with it. Clarify if you don't want people to "argue" with you.

And I don't consider it arguing when I debate. It's more like trying to get people to understand my point of veiw. *g*
Nasopotomia
06-01-2005, 11:02
Even if there is probably no God, then there is certainly no downside to believing in Him (unless, of course, it is a whacked-out cult God). Thus, A simple cost benefit analysis shows that it is better to believe in God.

Hey, guess what that is? Pascal's Wager with different wording. As your post, in essence, just was. Wow. Now, no offence intended, stuff it.

Yes, but any God who was happy with people taking a cost benefit analysis of faith and then turning to him and saying "I believed in you just in case you were real" isn't really one that I would worship. It's a silly penny-pinching sort of argument, the kind of thing I imagine lawyers and accountants using.. And I thought everyone agreed they'd be going to hell.


I bet you'd say the same to someone who claimed they were in love. How do you know you are in love? You cannot. It just happens. The same is true with believe in God. Do not scoff at someone's experiences - if someone told me they didn't believe in God because all their relatives and friends just died, I'd respect that (provided they weren't feeding me bull). Show a tad of common courtesy - logic only proves that it is better to treat people better, because you get far more in return.


Man, I'm not the one to argue with. Argue with that long-dead French philosopher, it's his quote, not mine. Besides, I said that it leads often to God, not always. You can't read my posts quickly, or you miss alot.

You must remember that plenty of long-dead philosopher's arguments weren't all that good. Descartes essentially said God must exist because everything was there, and everything was there because God exists. Now I'm not going to say Descartes was a fool, or anything like that; it's his system that science is entirely based on, after all, but to use a blatant tautology on what is perhaps one of the most important of all Philosophical questions definitely speaks to me of a bad day.


Hey, guess what? If you actually bothered to read my post, you would see the reference to "Monday morning" and "my philosophy teacher". I see the fallacies in Pascal's Wager, it's not why I believe in God. That was a joke - learn to see humour. Yeesh.

Yeah, but some of us (clearly not Dakini, but those of us READING your posts) are just saying why the wager can't stand up as a reason for faith.

My philosophy teacher is a genius - he was raised as an atheist, and converted to Catholicism at the age of 21. He has a way of making things make sense, even if it does get a bit fuzzy at times. I think I'd probably be his protege.

Is that a bit of vanity, or a pipe dream? :p

As to your Catholicism, do you actively follow scripture or do you follow the ideas behind it? Do you believe in the Ark, or the message that the story of the Ark puts across?

I personally view religion as a vital stage of cultural development, but not actually true in itself; the best analogy I can find for it is a "species-wide Santa Claus", but that's a little more flipant and desultory than I would like. The concept is that religion creates a false 'Big Brother' figure, to steal a little of Orwell, in order to explain the unexplainable but, far more importantly, to maintain order and control over any population over about 200 individuals. Hence, the huge importance placed on "Thou shalt not", the essential basis of the ten commandments.

This leads to my actual point. Could it not be that, while religion does have the correct conclusions (don't kill people), it's reasoning was regarded as almost completely unimportant as long as the end was achieved? And, if so, would that not make religion itself unethical in it's basis, despite it achieving a positive end, and also outmoded and deeply inflexible?


Actually, I tend to find God's presence much stronger in nature when I'm searching for Him than Mass when I am not (sometimes of one, sometimes of the other). I actually view that perfection as one of my own personal proofs of God - some of my best ideas have come from nature, and discussion with others therein.

I would beg your definition of God, before I comment overly on that. However, regarding the perfection of nature, you can approach it from a scientist's "evolutionary" point of view, too: Human's are, essentially, designed to be perfect for their environment, or as perfect as possible, as that's the point of evolution. Hence, evolution would also ensure that humans find nature pleasing, as they're designed to exist in it?
UpwardThrust
06-01-2005, 13:35
ah, but they're only visible in the 5th dimension and they can enter into our three dimensions when they please, appearing pink in both places. however, when they are only observing our world, they are invisible.

duh.
While flippant it is generally what happens with a lot of religious beliefs when challenged (either threw new evidence or what not) they make up a situation that is out of “known” bounds
E B Guvegrra
06-01-2005, 14:38
While flippant it is generally what happens with a lot of religious beliefs when challenged (either threw new evidence or what not) they make up a situation that is out of “known” boundsMy usual 'scientific' answer to "How can it be Pink and Invisible" is that its hide has the requesit 'pink' pigments that just happen (like the rest of the unicorn) to be somehow intangible to the electromagnetic waves we all know and love.

(BTW, here's an interesting fact I heard a while ago: There are green insects, green birds, green fish, green just about everything but no (intrinsically) green mammals. Tree sloths look green, but that's the moss and algae growing on their fur, so slowly they move around.)


And my answer to the original topic: Very similar to others' interpretations, is that I do not choose to not believe but rather have no reason to choose to be swayed by the arguments of belief. And, as others have said, the concept of 'choice' is a little difficult to justify. I'm a bit of a fatalist anyway, but even disregarding the whole predestination thing, I consider that my parallel-universe self who is a believer in something or other didn't go "You know, that sounds about right, I'm going to be swayed towards religion X", but was moulded by circumstance towards being a 'nurtured believer'. And while there may be some people who are 'natural believers', it is my opinion that nurture plays a much larger part and upbringing (RC convent education, Islamic school, Buddhist temple, a secular environment) encourages (or discourages) the growing child to follow the relevant type/lack of belief much more than either ancestry or concious decision does.
Pythagosaurus
06-01-2005, 15:15
Even if there is probably no God, then there is certainly no downside to believing in Him (unless, of course, it is a whacked-out cult God). Thus, A simple cost benefit analysis shows that it is better to believe in God.

Hey, guess what that is? Pascal's Wager with different wording. As your post, in essence, just was. Wow. Now, no offence intended, stuff it.

Congratulations. You got my reference. Now go back and read my post before the one you quoted.

I bet you'd say the same to someone who claimed they were in love. How do you know you are in love? You cannot. It just happens. The same is true with believe in God. Do not scoff at someone's experiences - if someone told me they didn't believe in God because all their relatives and friends just died, I'd respect that (provided they weren't feeding me bull). Show a tad of common courtesy - logic only proves that it is better to treat people better, because you get far more in return.

Are you suggesting that love is somehow logical? While I agree whole-heartedly that people should be nice to each other, I tend to reserve it for people who act according to what they say, and I never resort to direct attacks. For example, if you're going to criticize people for being illogical, then you should proceed to make a logical argument. If you insist that people should be courteous to each other, then you shouldn't tell them to stuff it. Capisce?
Bill Mutz
06-01-2005, 15:40
If you mean Christianity, I never said that outright. There are two other large monothestic religions, and they share the same God, just under a different name. So do all 3 depress you then.Islam is actually worse, and Judaism is, well, Judaism. I've got a Jewish boyfriend, and he's one of the many Jews I've known who don't really take the religion seriously. They think of themselves more as a race than as a religion. After all, Israel has among the highest percentages of nonbelievers in the world from what I've heard. Besides, I'm not going to go fleeing under the skirts of some other religion just because I found the one I was raised with unsatisfactory. I have too many other interests to really waste my time with it. Quite simply, I was an unsatisfied customer, so I gave it up. Also, I find it more satisfying to deal with things hands-on. All this aside, I believe that the institutions built around most if not all major religions are irreversibly corrupt, and I refuse to associate myself with them in any way.

Mostly, however, I just think that it's a silly belief for silly people.

I must agree with the sentiments of one poster who said that the religion just doesn't feel right. I kind of feel the same.

Lastly, when you get right down to it, I just can't bring myself to believe in a god who insists upon being admired all the time. Sounds like a Richard Cranium to me.
Bitchkitten
06-01-2005, 15:44
I think most of the logical reasons why I don't believe in god have been covered here. I thought perhaps the author of this thread might better appreciate the more emotional reasons that led me to start questioning in the first place. I grew up in the Bible Belt and the version of religion there was rather unpalatable. Mainly Southern Baptist. :gundge: I found them hateful, narrow-minded and just all around creepy. They said gays were abominations. Everyone who disagreed with them was going to hell. Women should be subservient to men. Their holy book said one thing and they did another. But being an *sshole was all right 'cause they were saved and Jesus forgave them. Why? Becuase they believed and worshipped the 'right' way. That's why I started looking for something different.
Faithfull-freedom
06-01-2005, 16:18
What leads people to believe there is no God(s)?

I think it stems from every religion stating God is just this one way. Like God has any formalities (limits) or expects any of our illusionary boundaries to really effect our lifes. I believe it comes down to all of the religions having one common denominator and the rest of thier bullshit is just that: bullshit. Faith is a bond that is needed. With absolute faith comes absolute freedom.
Eurotrash Smokey
06-01-2005, 16:25
The bible is just crappy catholic propaganda. killing all the muslims in the crusades was necessary too i think :confused:
Hakartopia
06-01-2005, 17:23
It's a question that has puzzled me because when I talk to various people that don't, they seem to have fairly different reasons, so you can't connect the dots. I'm simply curious why because believing or not believing in God is a concious choice we all make unless you're agnostic, so why folks?

I have never seen any evidence of a god, nor do I have need of one.
Neo Cannen
06-01-2005, 17:46
I'm an atheist.

Modern science has given such a successful explanation of just about everything that any necessity for a supreme being is totally gone. Need something to create humans? We've got evolution. Life? Abiogenesis. The Universe? Big Bang and inflationary cosmology, though I'll admit we're still working on that one. Since there's not any kind of logical need for the supernatural (i.e. there's nothing out there that can't be explained in a rational manner-even the stuff science hasn't really gotten laid out yet is expected to become known sooner or later), we don't need any gods. That, plus my own total lack of any experience leading me to believe, is proof enough for me that the Universe gets along just fine without any invisible men in the sky.

1) Abiogenesis has yet to be scinetificly proven

2) The idea of God does not originaly come from a lack of explination to describe the world. If that were true, what is Jesus for?
Neo Cannen
06-01-2005, 17:49
The bible is just crappy catholic propaganda. killing all the muslims in the crusades was necessary too i think :confused:

Ah, another uneducated person. I will point this out again and again untill someone realises it. The Crusades were as much a religious conflict as the second world war was an ideological conflict. They both had ideologies fighting (Christianity vs Islam and Fascism vs Capitalism/Communism) but the primary cause of the war was not religous/ideological. The cause was territorial. The Europeans had Jeresulum, the Saracans took it from them and the Europeans wanted it back. The Germans took free Europe (or large parts of it) and free Europeans wanted it back. While religion/ideology fuelled the war, it was not the cause.
Stephistan
06-01-2005, 17:57
It's a question that has puzzled me because when I talk to various people that don't, they seem to have fairly different reasons, so you can't connect the dots. I'm simply curious why because believing or not believing in God is a concious choice we all make unless you're agnostic, so why folks?

I think of course without doubt it's a conscience choice, however if you are raised in a family that is either rabid believers or rabid atheists, than I suppose you must add in quite a bit of social conditioning.

I myself was raised a Catholic, we went to church on Sundays. However my parents were not rabid about religion and thus I had a pretty open mind. When I could think it over for myself after reading the bible and other things, it just made no sense.In fact it made so little sense it was embarrassing. I basically came to the conclusion if there was a higher being it certainly wasn't the one these books were trying to sell me.

Then a time after I learned a lot about biology and the make up of the human being compared to other life on this planet and decided that nature was it's own higher power and certainly it would appear that nature does as it wants without any hand guiding it. Thus I no longer believed it was possible from an intellectual stand point for their to be any type of higher being. Thus, today I am an atheist.

I certainly have my opinions on why religion happened and why the people who run churches and religions do.. but that's for another topic.
Drunk commies
06-01-2005, 18:03
1) Abiogenesis has yet to be scinetificly proven

2) The idea of God does not originaly come from a lack of explination to describe the world. If that were true, what is Jesus for?
1 No they have not, but they have made discoveries that suggest it's possible

2 Jesus doesn't have much of anything to do with the origin of beleif in god. His birth and the origin of theism are separated by millenia. It's quite possible religion began as an effort to explain and control forces beyond human knowledge at the time. Once that gets the ball rolling beleivers keep it in motion and we get the current massive diversity of religious beleif. Kind of like evolution.
Pythagosaurus
06-01-2005, 18:11
1) Abiogenesis has yet to be scinetificly proven

2) The idea of God does not originaly come from a lack of explination to describe the world. If that were true, what is Jesus for?
1. Proving things is not the purpose of science. That would be developing theories that best fit the facts. Right now, the scientific community prefers abiogenesis over creationism. It doesn't matter whether or not it's been proven. If you poke legitimate holes in a scientific theory, then we develop better theories. We want you to try to disprove abiogenesis. That's the beauty of the scientific method, and it's also why religion can't be discussed in a scientific setting.

2. The idea of God does not originally come from the time of Jesus. By the time Jesus was around, religion had turned into a tool that people used to control the masses.
Santa Barbara
06-01-2005, 18:52
First of all: I do realise that there are some atheists/agnostics with decent arguments, who have taken philosophy - just like there are some theists out there who've taken philosophy also, myself included. There are exceptions. But look back on this thread - just how many people have made coherent, sensible, well-argued arguments? A handful at best.

So what, atheist/agnostics are stupid? There are incoherent, nonsensible, poorly argued arguments coming from all sides of the theological spectrum here. It's a general forum with mostly young teens. Whats your point?


Even if there is probably no God, then there is certainly no downside to believing in Him (unless, of course, it is a whacked-out cult God). Thus, A simple cost benefit analysis shows that it is better to believe in God.

Right, but I could make the same argument in favor of believing anything. What's the downside to believing the earth is flat? Your friends will make fun of you? Hell they could do that for you believing in God too. But I'm not going to "believe in God" simply because there aren't serious physical penalties for it. Nor would I believe the earth is flat.

I bet you'd say the same to someone who claimed they were in love. How do you know you are in love? You cannot. It just happens.

Hey, I saw the Matrix too, but I'm going to disagree with both you and The Oracle on that one. There are plenty of ways to tell you're in love - otherwise you wouldn't "claim" it at all. Just answer questions, like "do you feel attracted to this person, emotionally/sexually", "do you care for this person's well-being," etc. Love isn't some mystical thing that can't be defined or even recognized.

Do not scoff at someone's experiences - if someone told me they didn't believe in God because all their relatives and friends just died, I'd respect that (provided they weren't feeding me bull).

Hey I'll scoff at people's experiences if I damn well please! Especially if they're trying to convince me to join their club, as so many people who relate their religious/spiritual anecdotes to me are.


Actually, I tend to find God's presence much stronger in nature when I'm searching for Him than Mass when I am not (sometimes of one, sometimes of the other). I actually view that perfection as one of my own personal proofs of God - some of my best ideas have come from nature, and discussion with others therein.

Well, I find God nowhere, whether searching or not. Same with perfection. Same with actually-convincing pro-God arguments on message boards. :)
La Terra di Liberta
06-01-2005, 18:56
The bible is just crappy catholic propaganda. killing all the muslims in the crusades was necessary too i think :confused:



Catholics didn't write the Bible. They were the original chruch but when things like the crusades and what not happened, people began to question them (Martin Luther, John Calvin) and now Protestants exist. The crusades were lies and bs and a waste of human life and the modern church does not support what happened them as a good thing.
La Terra di Liberta
06-01-2005, 18:57
I think of course without doubt it's a conscience choice, however if you are raised in a family that is either rabid believers or rabid atheists, than I suppose you must add in quite a bit of social conditioning.

I myself was raised a Catholic, we went to church on Sundays. However my parents were not rabid about religion and thus I had a pretty open mind. When I could think it over for myself after reading the bible and other things, it just made no sense.In fact it made so little sense it was embarrassing. I basically came to the conclusion if there was a higher being it certainly wasn't the one these books were trying to sell me.

Then a time after I learned a lot about biology and the make up of the human being compared to other life on this planet and decided that nature was it's own higher power and certainly it would appear that nature does as it wants without any hand guiding it. Thus I no longer believed it was possible from an intellectual stand point for their to be any type of higher being. Thus, today I am an atheist.

I certainly have my opinions on why religion happened and why the people who run churches and religions do.. but that's for another topic.



Finally, someone that thinks it's a choice.........
Bob-omb
06-01-2005, 19:00
I'm agnostic because I don't want to believe in a God, because if he's real; I hate him.
Chizzilla
06-01-2005, 19:03
Look I'm gonna be frank: there is absolutely no proof that God exists or doesn't, so stop debating it. Religious people are crazies, but so are atheists. Live with the fact that religion is a man-made institution.
International Terrans
06-01-2005, 19:05
Sorry for the long post, people.

Congratulations. You got my reference. Now go back and read my post before the one you quoted.
I did, I merely did not have time to reply to it. And I still don't. When somebody blatantly tries to mock me, I'm going to reply right back.

Are you suggesting that love is somehow logical? While I agree whole-heartedly that people should be nice to each other, I tend to reserve it for people who act according to what they say, and I never resort to direct attacks. For example, if you're going to criticize people for being illogical, then you should proceed to make a logical argument. If you insist that people should be courteous to each other, then you shouldn't tell them to stuff it. Capisce?
Some of us don't have hour upon hour to waste on forum posts. That one alone took me 15 minutes. Believe me, at 1 in the morning and with school the next day, I simply don't have time for more. I'm even making this during my third period class.

I did make logical arguments - merely the ones that you so tactly ignored.

When someone is not courteous to me, such as directly laughing at something I said, then I will not be courteous in return. If you're a pot, don't call the kettle down the counter black.

The existence of God is not something that can be proved or unproved by logic, which is why you have no place here. The only thing that really counts are those testimonials that you so scoffed at. The arguments both for and against are by nature limited, and are completely inexclusive. I don't like Pascal's Wager, I merely respect it as an alternative (however flawed). Cost benefit analysis is the worst thing in the world, because human beings are, at their core, extremely irrational.

Belief, and unbelief, is not based in logic as much as people may claim it is.

First of all, I am no man,(I hate it when people assume that), secondly, if you don't stand by the quote then why did you post it? I did read your post thourghly, and I saw the "often", and I understood it too. What got me was the quote itself. I'm "arguing" with you because you posted it and didn't say anything against it, so that led me to believe that you agreed with it. Clarify if you don't want people to "argue" with you.
Saying "man" in that context does not designate gender. I don't know why you took that the wrong way; I say that to females all the time and they never care - you're the first person to object.

Know why I didn't respond? Because I didn't have enough time. I explained this earlier. I merely took where you were coming from, and although I disagree very much, I didn't care to create a flame.

Yes, but any God who was happy with people taking a cost benefit analysis of faith and then turning to him and saying "I believed in you just in case you were real" isn't really one that I would worship. It's a silly penny-pinching sort of argument, the kind of thing I imagine lawyers and accountants using.. And I thought everyone agreed they'd be going to hell.
It was a joke in response to a joke. Believe me, God hates such a thing. Even Pascal said that it was just for people to think about the benefits of faith; faith does not immediately follow. I'd say the same sort of thing about an insincere "deathbed conversion".

Is that a bit of vanity, or a pipe dream?
Well, I'm the one of his students (in the past few years) that most closely resembled him, although we do have many differences. It's true, but not very much.

As to your Catholicism, do you actively follow scripture or do you follow the ideas behind it? Do you believe in the Ark, or the message that the story of the Ark puts across?
No, I'm more of an message person. Scripture in the Old Testament and Revelations to me should be taken more for it's message, and not the literal translation. For your example, I'd go with the message, but did it ever occur that the Red Sea might once have been a depression and not a sea? ;)

This leads to my actual point. Could it not be that, while religion does have the correct conclusions (don't kill people), it's reasoning was regarded as almost completely unimportant as long as the end was achieved? And, if so, would that not make religion itself unethical in it's basis, despite it achieving a positive end, and also outmoded and deeply inflexible?
However, those deep-seated "correct" conclusions were actually brought to you care of religion. Without religion, they would disappear - as it has been so in every past society without a strong moral religion (for instance, the Huns and the Mongols). Religion may have commited such mistakes in the past, but we've done our penence, and are ready to banish our history of such mistakes. As the things you're talking about are now past, and our reasoning is far more important, it is now a moot point.

I would beg your definition of God, before I comment overly on that.
My definition? My view of God is probably closer to the Native American idea of the "Great Spirit" than to the personified God that many Christians believe in. In short, I'd view Him more along the lines of a great force, beyond true human comprehension, with unlimited knowledge, unlimited power, and an ineffable plan that we're just going to have to go along with. But, like I said, God is beyond human comprehension (the one part I'm sure about), so I could be wrong.

I think most of the logical reasons why I don't believe in god have been covered here. I thought perhaps the author of this thread might better appreciate the more emotional reasons that led me to start questioning in the first place. I grew up in the Bible Belt and the version of religion there was rather unpalatable. Mainly Southern Baptist. I found them hateful, narrow-minded and just all around creepy. They said gays were abominations. Everyone who disagreed with them was going to hell. Women should be subservient to men. Their holy book said one thing and they did another. But being an *sshole was all right 'cause they were saved and Jesus forgave them. Why? Becuase they believed and worshipped the 'right' way. That's why I started looking for something different.
Those are Southern Baptists. I never understood the reason why, if one denomination has bad sides, all the others do as well, although you obviously have other reasons. We're not all Protestant fundamentalists.

Well, that was a waste of my anthropology class. Oh well.
Nhetsmm
06-01-2005, 19:13
I'm agnostic.

Perhaps your question really is "why don't I believe in your God?".


I agree. I am very spiritual, just not religious. I have ideas about supreme beings and the afterlife, but none are the same as anyone else's.
Nhetsmm
06-01-2005, 19:21
Look I'm gonna be frank: there is absolutely no proof that God exists or doesn't, so stop debating it. Religious people are crazies, but so are atheists. Live with the fact that religion is a man-made institution.

Ah, the 'opiate of the people' argument. Good point
Pythagosaurus
06-01-2005, 19:29
When someone is not courteous to me, such as directly laughing at something I said, then I will not be courteous in return. If you're a pot, don't call the kettle down the counter black.
Meh. This has to stop somewhere. I'm not too big to admit fault.

The existence of God is not something that can be proved or unproved by logic, which is why you have no place here.
I never said it could. In fact, several times in other threads, I have pointed out that precise fact. I don't see what any of this has to do with my presence in this thread. After all, the thread asks people why they don't believe in God. So, if you belong here, then why don't you believe in God?

If you'll look back, you'll see that I've never tried to argue the inexistence of God. I've merely set up a parallel between the belief in God and the belief in pink elephants. I'm not suggesting that pink elephants don't exist or that one shouldn't believe in them, only that they're equally rational compared to a belief in God. Further, if I did manage to convince somebody that this is true, then I would not use it to argue that the person shouldn't believe in God. I would use it to argue that there's no rational reason for (or, equally, against) a belief in God; people believe because they do. There's no conscious choice, only a conscious realization.
Grave_n_idle
06-01-2005, 19:34
1) Abiogenesis has yet to be scinetificly proven

2) The idea of God does not originaly come from a lack of explination to describe the world. If that were true, what is Jesus for?

1) Okay, Neo - Abiogenesis has yet to be proven, unlike the idea of some big floaty guy making planets in six days... for which we see new evidence daily? Nobody has claimed that Abiogenesis has been proven... but, when you follow all the scientific observation, it gives you certain ideas of how our world works, which gives you certain ideas of how the world WORKED.

2) You may not have noticed... religion got on quite happily for at least 23,000 years (basing from the Venus of Willendorf) BEFORE Jesus appeared on the scene - which means he is hardly 'required' for religion to work.

At the most base level, all primitive religions seek to explain elemental phenomena, and seek to find ways to placate disaster - leading to anthropomorphic representatives of the elements, etc. Which is why primitive religions are almost always polytheistic... which is even evident in Judaism, if you read the Old Testament in the original Hebrew.

AT later levels, people begin to texture their beliefs, sometimes by exposure to other cultures, sometimes, by trying to explain things that can't be explained under the current model.... sometimes just for fun, maybe.

Look at the Roman/Greek pantheon... Gods having sons with human women... not that unusual - even the Old Testament shows evidence of godlike beings spawning offspring on human women.

Jesus doesn't enter the mythology for a thousand years after most of it was written, and if you are LOOKING for a reason 'why' he enters the story, you have to look no further than the Prophets, who had predicted a direct intervention by god, that never seems to have arrived.
Uldaedia
06-01-2005, 21:45
Saying "man" in that context does not designate gender. I don't know why you took that the wrong way; I say that to females all the time and they never care - you're the first person to object.

Know why I didn't respond? Because I didn't have enough time. I explained this earlier. I merely took where you were coming from, and although I disagree very much, I didn't care to create a flame.

Sorry, but where I live it does.
What did I say about respnding? I meant you should have clarified in your post if you were against or for it or what. That way people like me won't get all upset. If you don't have time to write the whole post then don't post it. An wuthor wouldn't publish only half of the book.

Basically (and I know somebody said this before, applause for them) I get tired of people telling everyone that religion is good for you and fighting over whether or not pink unicorns exist. I have nothing against religious people, but the same thing does not work for everyone. Some people just aren't the religious type, and whether or not there is a God isn't going to make a difference.

By the way, Mr. Philosophy? The definition of "philossophy" is the study of the basic ideas about knowledge, truth, right and wrong, God, and the nature and meaning of life. I find it really funny that you keep telling all of these people to take a philosophy course before they argue. Philosophy is about your own ideas about the subjects listed above, looking for answers, however odd they may be. And some of them are odd. I find the story of an all-powerful man making the world in seven days and then creating life really odd. The subject is not like science. It's not exact.

Anyway, I think I'll start a sister thread to this one. I'm interested to see what people will say.